Rheumatology RACP MCQs Flashcards

1
Q

What is the greatest predictor of future thrombosis in patients with anti-phospholipid syndrome?
A. Annexin Ab
B. Anticardiolipin Ab
C. Lupus anticoagulant
D. Beta 2 glycoprotein 1 Ab
E. Anti-prothrombin Ab

A

C Lupus anticoagulant

The presence of LA or triple aPL positivity are the main risk factors for recurrence.

How well did you know this?
1
Not at all
2
3
4
5
Perfectly
2
Q

A 27 year old lady with SLE presents with recurrent miscarriage. Which antibody predicts further miscarriages?
A. Anti-SSa
B. Anti-histone
C. Anti-cardiolipin
D. Anti-dsDNA
E. Anti-Smith

A

C - Anticardiolipin

History suggestive of obstetric APS -defined by recurrent miscarriages, late (>10 wks) foetal demise and premature births due to eclampsia/pre-eclampsia and placental insufficiency. Lupus anticoagulant, anti cardiolipin ab and beta2glycoprotein1 ab are characteristic of APS.

How well did you know this?
1
Not at all
2
3
4
5
Perfectly
3
Q

In a 75-year-old woman presenting with a unilateral temporal headache, which of the following clinical features is most specific for a diagnosis of temporal arteritis?

A. Temporal artery tenderness.
B. Blurred vision.
C. Jaw (masseter) claudication.
D. Pain and stiffness around the shoulders and hips.
E. Fever.

A

C. Jaw (masseter) claudication.

50% of patients with GCA have jaw claudication and is the feature most highly associated with a positive biopsy for GCA.

How well did you know this?
1
Not at all
2
3
4
5
Perfectly
4
Q

Which of these features has the highest likelihood ratio for giant cell arteritis (GCA)?

A. Headache
B. Diplopia
C. Jaw claudication
D. Joint synovitis
E. Proximal myalgia

A

C. Jaw claudication

How well did you know this?
1
Not at all
2
3
4
5
Perfectly
5
Q

Which of the following cells is most important in the pathogenesis of vasculitis in temporal arteritis?

A. Multinucleate giant cell
B. Macrophage
C. CD8+ T cell
D. CD4+ T cell
E. Dendritic cell

A

C. CD4+ T cell

Is the cell that initiates differentiation and activation of macrophages and the subsequent granulomatous inflammation.

How well did you know this?
1
Not at all
2
3
4
5
Perfectly
6
Q

A 72-year-old man presents with a right sided headache for two weeks. On examination he is tender over the temporal artery. His erythrocyte sedimentation rate (ESR) is 82 mm/hr [0-22].

Which is the most appropriate next investigation?
A. Ultrasound of temporal artery.
B. Temporal artery biopsy.
C. Platelet count.
D. Retinal photography.
E. Serum protein electrophoresis.

A

B. Temporal artery biopsy.

CDUS of the head, neck, and upper extremities can serve as a diagnostic surrogate for temporal artery biopsy when performed by clinicians skilled in this technique. In the absence of extensive experience with this technology in routine clinical practice, however, temporal artery biopsy remains an essential diagnostic measure for the evaluation of suspected GCA.

How well did you know this?
1
Not at all
2
3
4
5
Perfectly
7
Q

A 70-year-old woman, weighing 45 kg, presents with polymyalgia rheumatica and headache. There is a history of osteoporosis with a previous vertebral wedge fracture. Past history also includes hysterectomy.
Temporal artery biopsy reveals giant cell arteritis. A bone mineral density test shows T scores for the lumbar spine and the femoral neck as -3.0 and -2.5 respectively. A chest X-ray shows wedging of the thoracic vertebra on the lateral view.

The most appropriate initial management is:

A. prednis(ol)one 40 mg daily and alendronate 10 mg daily.
B. prednis(ol)one 10 mg daily and azathioprine 100 mg daily.
C. prednis(ol)one 40 mg twice daily and calcitriol 0.25 μg twice daily.
D. prednis(ol)one 15 mg daily and methotrexate 10 mg weekly.
E. prednis(ol)one 40 mg daily and hormone replacement therapy

A

A. prednis(ol)one 40 mg daily and alendronate 10 mg daily.

GCA treatment - high dose glucocorticoids (40-60 mg) daily

First line treatment of osteoporosis, esp. that caused by steroids - alendronate

How well did you know this?
1
Not at all
2
3
4
5
Perfectly
8
Q

A 35 yo woman presents with a 3-day history of headaches and vomiting. She was diagnosed gastrooesophageal reflux disease 12 months ago and Raynaud’s henomenon 2 years ago. On examination her blood pressure is elevated at 225/145 mmHg, her pulse is 80/min. She has changes of sclerodactyly affecting her fingers, dorsum of hands and has evidence of thickened, tight skin over the anterior chest wall.

Relevant investigations are shown below
Hb 110 g/L
WCC 8.5 x 109/L
Platelets 75 x 109/L
Blood film Schistocytes, low platelets
Urea 10 mmol/L
Creatinine 140 umol/L (eGFR 42 ml/min)
U/A: Protein ++, blood and other markers all –ve

Which of the following is the most appropriate treatment?
A. Captopril
B. Sodium nitroprusside
C. Pulse methylprednisolone
D. Cyclophosphamide

A

A. Captopril

Patient has scleroderma renal crisis - hypertensive crisis, thrombocytopenia, MAHA (micorangiopathic haemolytic anaemia) and proteinuria.

Treatment:
- Rapid correction with Captopril (this is the ACEi of choice because of extensive clinical experience and its short onset and duration of action, which permit rapid dose escalation)
- Intravenous Nitroprusside in ADDTION if there is CNS symptoms
- Dialysis ( 2/3 patients require this)
- Eculizumab (complement pathway antagonist)
- Renal transplant (if unable to wean dialysis >2 yrs)

How well did you know this?
1
Not at all
2
3
4
5
Perfectly
9
Q

In systemic sclerosis, which of the following autoantibodies is associated with scleroderma renal crisis?

A. Anti-centromere
B. Anti-topoisomerase
C. Anti-RNA polymerase III
D. Anti-Smith

A

C. Anti-RNA polymerase III - assc. with diffuse SSc and scleroderma renal crisis

  • Anti-centroemere - specific for localised SSc and pulmonary artery hypertension
  • Anti-topoisomerase (Anti Scl 70) - specific for diffuse SSc but associated with ILD
    Anti-Smith - associated with SLE and MCTD
How well did you know this?
1
Not at all
2
3
4
5
Perfectly
10
Q

A 64yo lady presents with sicca symptoms. She has also noted difficult with dorsiflexion of
her right foot, and a tingling sensation in her fingers.
Investigations show :
ANA : 1: 2560
ENAs : positive for SSA(Anti-Ro) and SSB (Anti-La) , negative for dsDNA
RF 85

What is the diagnosis?
A. Rheumatoid arthritis
B. SLE
C. Primary Sjogrens syndrome
D. Overlap syndrome
E. Scleroderma

A

C. Primary Sjogrens syndrome

ANA - positive in SLE, RA and Sjogrens
Anti Ro/SSA and Anti La/SSB positive in SLE and Sjogrens
dsDNA - specific for SLE
RF positive in RA, SLE and Sjogrens

How well did you know this?
1
Not at all
2
3
4
5
Perfectly
11
Q

RACP 2022a Q4

65F with background of Sjogren’s disease. New rash on the lower limbs
(photo given, purpuric, macular, irregular shaped patches mostly <1cm x 1cm in each section, did not look urticarial; looked vasculitic). Associated with loss of sensation over the foot. Investigations list: ANA/SSA/SSB Positive; RF
300; ACPA Neg; p ANCA weak positive. MPO/PR3 neg. HIV/HepB/C negative. Complement (3/4) both low.

a. Hypocomplimentaemic urticarial vasculitis
b. rheumatoid vasculitis
c. mixed cryoglobulinemia
d. microscopic polyangiitis

A

Answer D Microscopic polyangitis

How well did you know this?
1
Not at all
2
3
4
5
Perfectly
12
Q

RACP 2022a Q9
45 year old man developed myopathy on atorvastatin 6 months ago. His CK was normal at the time and his symptoms improved after cessation of the statin. He now presents with worsening proximal myopathy with a CK of
4000. Muscle biopsy demonstrates necrosis, macrophage infiltration and MH1 staining. What is the most likely cause?

a. Immune mediated necrotic myositis
b. Statin toxic myopathy
c. Inclusion body myositis
d. Polymyositis

A

Answer A IMNM

Initial reaction to statin was likely toxic myopathy

Following this - current reaction with elevated CK, necrosis, macrophage infiltrate and MHC I staining in keeping with immune mediated necrotising myopathy - caused by abs against HMGCR

How well did you know this?
1
Not at all
2
3
4
5
Perfectly
13
Q

RACP 2022a Q13

13.A 56 year old woman has autoimmune hepatitis, hypertension and hyperlipidemia on azathioprine, perindopril and atorvastatin. She develops acute joint pain in her left MTP. Joint aspirate showed calcium pyrophosphate (?) crystals. What is your initial management?

a. Allopurinol
b. Probenicid
c. Feboxustat
d. Benzbromarone

A

Answer B Probenecid

Allopurinol and febuxostat interfere with the metabolism of azathioprine, increasing plasma levels of 6-mercaptopurine which may result in potentially fatal blood dyscrasias.

Benzbromarone reduces hyperuricaemia by blocking renal tubular reabsorption of uric acid. It is also hepatotoxic.

How well did you know this?
1
Not at all
2
3
4
5
Perfectly
14
Q

RACP 2022a Q34

34.What is skin lesion associated with antiphospholipid syndrome?

a. Livedo reticularis
b. Digital ulcers
c. Erythema annulare
d. Palpable purpura

A

Answer A

Livedo is the most common cutaneous manifestation of APS. Although livedo reticularis is nonspecific, livedo racemosa can be associated with arterial lesions and multiple thromboses in APS.

How well did you know this?
1
Not at all
2
3
4
5
Perfectly
15
Q

RACP 2022a Q64

64.Testing for which antiphospholipid antibody is affected by warfarin administration?

a. Lupus anticoagulant
b. Anti-cardiolipin
c. Anti-beta2glycoprotein
d. Protein C

A

Answer A Lupus anticoagulant

aCL and anti-beta2GPI antibody results are unaffected by anticoagulation. By contrast, different anticoagulants have variable effects on laboratory tests used to identify LA, making interpretation of the results more challenging.

How well did you know this?
1
Not at all
2
3
4
5
Perfectly
16
Q

RACP 2022b Q25

  1. A patient on infliximab develops new skin lesions (repeat question - picture shown)

A. Subacute cutaneous lupus
B. Psoriasis

A

Answer B Psoriasis

The use of infliximab in rheumatoid and inflammatory bowel diseases (IBD) has been associated with a variety of adverse skin reactions, including paradoxical psoriatic lesions.
Not enough information recalled to answer question.

How well did you know this?
1
Not at all
2
3
4
5
Perfectly
17
Q

RACP 2022b Q 30

  1. Neonatal heart block is associated with antibodies anti-Ro/SSA and anti-La/SSB.
    What period of gestation does neonatal heart block develop?
    A. 12-18 weeks
    B. 18-24 weeks
    C. 24-30 weeks
    D. 30-36 weeks
A

Answer B 18-24 weeks

Congenital heart block is usually detected when the fetus is between 18 and 24 weeks old.

How well did you know this?
1
Not at all
2
3
4
5
Perfectly
18
Q

RACP 2022b Q59

  1. Neonatal heart block is associated with antibodies anti-Ro/SSA and anti-La/SSB.
    What period of gestation does neonatal heart block develop?
    A. 12-18 weeks
    B. 18-24 weeks
    C. 24-30 weeks
    D. 30-36 weeks
A

Answer B 18-24 weeks

Congenital heart block is usually detected when the fetus is between 18 and 24 weeks old.

How well did you know this?
1
Not at all
2
3
4
5
Perfectly
19
Q

RACP 2021a Q11

Q11. Which of the following features is most specific clinical sign for psoriatic arthritis?

A. Asymmetrical swelling of DIP joints
B. Dactilitis
C. Nail pitting
D. Enthesisits

A

Answer A

Distal arthritis and arthritis mutilans are considered most specific for PsA, but they are not the most common patterns seen

How well did you know this?
1
Not at all
2
3
4
5
Perfectly
20
Q

RACP 2020a Q41

  1. 32 yo female with 12 month history of fatigue, arthralgia, sterile dysuria, dry eyes/mouth and
    parotid enlargement. CRP: 8, ESR: 90. Had vasculitis screen which showed ANA of 1:640, ENA positive for Anti-Ro/SSa + Anti-La SSb. Rheumatoid factor: 540 and Anti-CCP: 12 (<10).
    Lymphopenia and raised globulins. What is the most likely diagnosis?
    A) MCTD
    B) RA
    C) Sjogrens
    D) SLE
A

Answer C Sjogrens

Anti-SSA/Ro and anti-La/SSB are the hallmark antibodies in primary Sjögren’s syndrome (pSS), being present in 60-70% of patients.

How well did you know this?
1
Not at all
2
3
4
5
Perfectly
21
Q

RACP 2020a Q77

  1. Methotrexate used in RA and other inflammatory diseases. Which enzyme does methotrexate
    inhibit ?
    A) Dihydrofolate reductase
    B) Dihyroorotate reductase
    C) Inosine monophosphate dehydrogenase
    D) Thiopurine methyltransferase
A

Answer A Dihydrofolate reductase

The inhibition of DHFR by folate antagonists (methotrexate) results in a deficiency in the cellular pools of thymidylate and purines and thus in a decrease in nucleic acid synthesis. Therefore, methotrexate interferes with DNA synthesis, repair, and cellular replication.

How well did you know this?
1
Not at all
2
3
4
5
Perfectly
22
Q

RACP 2020

  1. Recurrent pancreatitis. Enlarged lacrimal glands. Retroperitoneal lymph node enlargement with
    dilated bile ducts on imaging. A biopsy of lacrimal glands that shows storiform fibrosis.
    A) Sarcoidosis
    B) IgG4 disease
    C) Lymphoma
    D) Sjogren’s Syndrome
A

Answer B IgG4 disease

The fibrosis associated with IgG4-RD usually has a characteristic “storiform” pattern, typified by a cartwheeling appearance of the arranged fibroblasts and inflammatory cells. The majority of patients have elevated serum levels of IgG4.

How well did you know this?
1
Not at all
2
3
4
5
Perfectly
23
Q

RACP 2019a Q4

Question 4
A 35 year old Turkish lady has joint pain, oral and genital ulcers, red painful nodules on legs,
occasional painful red eye. What is the most likely diagnosis?
A. Behcet disease
B. Sarcoidosis
C. Systemic lupus erythematosus
D. Granulomatosis with polyangiitis

A

Answer A Behcets disease

Behcet disease is an auto-inflammatory systemic vasculitis of unknown etiology. It is characterized by mucocutaneous manifestations, including recurrent oral and genital ulcerations, ocular manifestations, especially chronic relapsing uveitis, and systemic vasculitis involving arteries and veins of all sizes. Tt is observed commonly among populations living along the historic Silk Road, an ancient trading route that spans from Japan and China in the Far East to the Mediterranean Sea, including countries such as Turkey and Iran.

How well did you know this?
1
Not at all
2
3
4
5
Perfectly
24
Q

RACP 2019 a Q10

Question 10
Which of the following disease causes predominance of upper lobe pulmonary fibrosis?
A. Rheumatoid arthritis
B. Diffuse scleroderma
C. Mixed connective tissue disease
D. Ankylosing spondylitis

A

Answer: D Ankylosing spondylitis

How well did you know this?
1
Not at all
2
3
4
5
Perfectly
25
Q

RACP 2019a Q38

Question 38
A 36 year old female personal trainer presents with 6 months of right heel pain, worse when weight bearing. Over the last few weeks, she has developed left heel pain. She has a cousin with SLE and her GP checked her ANA (positive with a 1:160 titre). Double-stranded DNA is negative.
What is the most appropriate management?
A. NSAID and physiotherapy
B. Entheseal steroid injection to plantar fascia
C. Hydroxychloroquine
D. Corticosteroid

A

Answer A

Diagnosis: plantar fasciitis

First line - physiotherapy
Second line: NSAIDs
Third line : steroid injection

How well did you know this?
1
Not at all
2
3
4
5
Perfectly
26
Q

RACP 2019a Q58

Question 58
A 40 year old lady is on hydroxychloroquine and prednisolone for SLE. What is the next best agent in
lupus patient with non-renal lupus?

A. Methotrexate
B. Mycophenolate
C. Azathioprine
D. Cyclosporine

A

Answer A or C

MTX or AZA can be used - MTX has better evidence , but contraindicated in pregnancy.

As this patient is 40yo - consider AZA?

https://ard.bmj.com/content/78/6/736

How well did you know this?
1
Not at all
2
3
4
5
Perfectly
27
Q

RACP 2019a Q86

A 62 year old lady presents with a violaceous rash over her face and biopsy confirms lupus pernio.
With which disease is this most commonly associated?
A. Sjogren’s syndrome
B. Sarcoidosis
C. Systemic lupus erythematosus
D. Psoriatic arthritis

A

Answer B Sarcoidosis

Lupus pernio is a rare cutaneous manifestation of sarcoidosis. It presents as a violaceous lesion or shiny nodules over the head and neck but is predominantly localized to the nose, cheeks, and ears. Lupus pernio is often a predictor of systemic sarcoidosis, which warrants thorough workup. Patients with lupus pernio have a higher risk for pulmonary disease,

How well did you know this?
1
Not at all
2
3
4
5
Perfectly
28
Q

RACP 2019a Q90

Question 90
A 25yo personal trainer has presented concerned about the possibility of lupus. Her sister was
recently diagnosed with lupus and her GP performed blood tests demonstrating an ANA of 1:160.
DsDNA and ENA were negative. She describes long standing right heel pain on weight-bearing. What
is the most appropriate next step in management?
A. NSAIDs and physiotherapy
B. Steroid injection
C. Prednisone
D. Hydroxychloroquine

A

Answer A

How well did you know this?
1
Not at all
2
3
4
5
Perfectly
29
Q

RACP 2019a Q99

EMQ 99 and 100
A. C6
B. C7
C. Median elbow
D. Median wrist
E. Radial forearm
F. Radial spiral
G. Ulnar elbow
H. Ulnar wrist
Question 99
A 45 year old bartender presents with severe clawing of the ring and 5th finger.

A

Answer H Ulnar nerve at wrist

Motor complaints may include weakness/paralysis of the intrinsic muscles of the hand innervated by the ulnar nerve, which may present as a weakening of the handgrip and clawing of the fourth and fifth digits. Hypothenar atrophy may be present in more advanced cases. Differentiation between ulnar nerve compression at Guyon canal (wrist) vs. cubital tunnel (elbow) can be done by testing the strength of intrinsic (hand) vs. extrinsic (forearm) muscles supplied by the ulnar nerve respectively. Sparing of the dorsal surface of ulnar dermatome (dorsal medial hand and two fingers) points to Guyon canal syndrome.

How well did you know this?
1
Not at all
2
3
4
5
Perfectly
30
Q

RACP 2019a Q100

EMQ 99 and 100
A. C6
B. C7
C. Median elbow
D. Median wrist
E. Radial forearm
F. Radial spiral
G. Ulnar elbow
H. Ulnar wrist

A 54 year old lady who is a data analyst presents with pain and tingling in her thumb and index finger while driving. This pain also wakes her at night. The symptoms last for minutes and improve with shaking of her hand.

A

D Carpal tunnel syndrome - median nerve affected at wrist

Numbness, tingling, burning, and pain — primarily in the thumb and index, middle, and ring fingers. This often wakes people up at night. Many patients find that moving or shaking their hands helps relieve their symptoms. Repeating the same hand and wrist motions or activities over a prolonged period of time may aggravate the tendons in the wrist, causing swelling that puts pressure on the nerve.

How well did you know this?
1
Not at all
2
3
4
5
Perfectly
31
Q

RACP 2019b Q24

Question 24
In psoriatic arthritis, methotrexate is most helpful for:
A. Small joint arthritis
B. Axial arthritis
C. Enthesitis
D. Nail psoriasis

A

Answer : ? A
MTX useful for polyarthritis with/ without dactylitis

https://ard.bmj.com/content/79/6/700.1

How well did you know this?
1
Not at all
2
3
4
5
Perfectly
32
Q

RACP 2018a Q1

Febuxostat is a newer urate-lowering therapy than allopurinol. Based on randomised controlled
trial data in chronic gout, what is the main benefit of febuxostat 80 mg daily over allopurinol
300 mg daily?
A. Faster resolution of tophi.
B. Fewer cardiovascular events.
C. Fewer flares of acute gout.
D. Less radiologic articular changes.
E. Lower serum uric acid.

A

Answer E

In comparison with allopurinol, febuxostat is a more selective xanthine oxidase inhibitor and more potent in lowering serum urate levels.

Also note: febuxostat associated with increased incidence of cardiac events and all cause mortality compared to allopurinol.

How well did you know this?
1
Not at all
2
3
4
5
Perfectly
33
Q

RACP 2018a Q34

34.A 72-year-old man presents with a 3-month history of malaise, arthralgias, abdominal pain, and left foot weakness. In addition to left foot drop, examination reveals that he is hypertensive and
has a non-blanching lace-like skin discolouration on his legs.
Laboratory findings:

Normal values
Erythrocyte sedimentation rate (ESR) 102 mm/hr [0–15]
C-reactive protein (CRP) 68 mg/L [< 5]
Creatinine 120 μmol/L [45–90]
MSU 1+ proteinuria
CT scan angiography shows multiple small vessel aneurysms in the renal and mesenteric
arteries.
What is his autoantibody screening most likely to show?

A. c-ANCA.
B. dsDNA autoantibodies.
C. No autoantibodies detected.
D. p-ANCA.
E. Rheumatoid factor.

A

Answer C

Stem is consistent with polyarteritis nodosa which is not associated with any particular antibodies.

How well did you know this?
1
Not at all
2
3
4
5
Perfectly
34
Q

RACP 2018a Q45

  1. An 83-year-old woman presents with increasing difficulty walking. She develops left buttock
    pain after walking approximately 700 m or standing for a prolonged period. The pain radiates to the left calf and is eased by leaning on a shopping trolley or resting for 5 minutes. She has no pain on sitting.
    What is the most likely diagnosis?
    A. Ankylosing spondylitis.
    B. Osteoarthritis of the hip.
    C. Peripheral arterial disease.
    D. Spinal canal stenosis.
    E. Vertebral fracture.
A

Answer C Spinal canal stenosis

Forward flexion of the back may open up narrowed spinal-canal foraminal spaces in patients with lumbar spinal stenosis and improve their pain symptoms. These patients describe pain relief when leaning forward to push a shopping cart (“shopping cart sign”)

How well did you know this?
1
Not at all
2
3
4
5
Perfectly
35
Q

RACP 2018a Q75

  1. An 86-year-old lady with hypertension, type 2 diabetes and osteoporosis is found to have mild primary hyperparathyroidism. Her usual medications are metformin, quinapril,
    hydrochlorothiazide, cholecalciferol and alendronate.
    In light of the new diagnosis, which of her medications should be discontinued?
    A. Alendronate.
    B. Cholecalciferol.
    C. Hydrochlorothiazide.
    D. Metformin.
    E. Quinapril.
A

Answer C: Hydrochlorothiazide

PTH stimulates osteoclast activity and increases serum calcium.

Hydrochlorothiazide causes increased tubular reabsorption of calcium and may cause hypercalcaemia in patients with hyperparathyroidism

How well did you know this?
1
Not at all
2
3
4
5
Perfectly
36
Q

RACP 2018b Q102

102.A 68-year-old man with rheumatoid arthritis for 15 years has the following bone densitometry
results obtained on a dual energy x-ray absorptiometry scan:
T score Z score
Lumbar spine +1.3 +2.0
Femoral neck −2.4 −1.7

What is the most likely explanation for the discrepancy between the femoral neck and lumbar
spine readings?
A. Metastatic bone disease.
B. Osteonecrosis of the hip.
C. Osteophyte formation in the lumbar spine.
D. Paget disease in the lumbar spine.
E. Synovitis in the hip.

A

Answer C Osteophyte formation in the lumbar spine

How well did you know this?
1
Not at all
2
3
4
5
Perfectly
37
Q

RACP 2018 Q125

125.If a patient has strong positive anti-dsDNA antibodies (> 100 IU/mL) and a negative screen for
extractable nuclear antigens (ENA), what is the most likely pattern on antinuclear antibody
(ANA) testing?
A. Centromere.
B. Homogeneous.
C. Negative (no pattern seen).
D. Nucleolar.
E. Speckled.

A

E Speckled

SLE can be speckled (52%), homogenous (30% ) and nucleolar

How well did you know this?
1
Not at all
2
3
4
5
Perfectly
38
Q

RACP 2018 Q128

128.Rheumatoid factor antibodies are directed against which antigenic component?
A. Citrullinated peptides.
B. Deoxyribonucleic acid (DNA).
C. Extractable nuclear proteins.
D. Neutrophil cytoplasmic antigens.
E. The Fc portion of immunoglobulin G.

A

Answer E
Rheumatoid factors are antibodies directed against the Fc region of immunoglobulin G

How well did you know this?
1
Not at all
2
3
4
5
Perfectly
39
Q

RACP 2018 Q129

129.What is the most prevalent neuropsychiatric complication of systemic lupus erythematosus?
A. Anxiety disorder.
B. Cognitive dysfunction.
C. Major depression.
D. Mania.
E. Psychosis.

A

Answer ?B

https://www.ncbi.nlm.nih.gov/pmc/articles/PMC4349748/#:~:text=The%20most%20frequent%20NPSLE%20manifestations%20are%20headaches%2C%20psychiatric%20disorders%20(depression,of%20SLE%20diagnosis%20%5B7%5D.

How well did you know this?
1
Not at all
2
3
4
5
Perfectly
40
Q

RACP 2018 Q134

134.What is the most common pattern of interstitial lung disease in rheumatoid arthritis?
A. Acute interstitial pneumonia.
B. Desquamative interstitial pneumonia.
C. Lymphocytic interstitial pneumonia.
D. Organising pneumonia.
E. Usual interstitial pneumonia.

A

Answer E

The most typical radiological pattern of RA-ILD is usual interstitial pneumonia (UIP), representing more than half of the cases, followed by nonspecific interstitial pneumonia (NSIP), organizing pneumonia (OP), and lymphocytic interstitial pneumonia (LIP)

How well did you know this?
1
Not at all
2
3
4
5
Perfectly
41
Q

RACP 2018 Q151

151.Patients should be considered for investigation for secondary causes of osteoporosis if their
z-score is less than −2.0.
Assuming normal distribution, what does a z-score of −2.0 indicate?
A. A patient with this bone mineral density is in the lowest 1% of age-matched peers.
B. A patient with this bone mineral density is in the lowest 2.5% of age-matched peers.
C. A patient with this bone mineral density is in the lowest 5% for age-matched peers.
D. A patient with this bone density is twice as likely to fracture than other women her age.
E. A patient with this bone density is twice as likely to have a secondary cause of osteoporosis
compared with a young female.

A

Answer C

Z score represents standard deviations compared to age and gender matched means.

Z score of -2 is 2 standard deviations below the mean. Usually 2 standard deviations encompasses 95% of values. Hence Z score < -2 represents a score in the lowest 5% for that population.

How well did you know this?
1
Not at all
2
3
4
5
Perfectly
42
Q

RACP 2018 Q163

163.Bisphosphonates primarily act on which cell?
A. Macrophage.
B. Osteoblast.
C. Osteoclast.
D. Osteocyte.
E. Pericyte.

A

Answer C

By specifically inhibiting osteoclast-mediated bone resorption, bisphosphonates presumptively allow bone-forming osteoblasts more time to promote bone formation, albeit in the setting of abnormal collagen matrix.

How well did you know this?
1
Not at all
2
3
4
5
Perfectly
43
Q

RACP 2017a Q11

A 75-year-old Chinese man who is being treated with a TNF inhibitor for Rheumatoid Arthritis
develops fevers, night sweats and a productive cough. You suspect Tuberculosis.The most sensitive test for diagnosing active TB infection is:
a) Three consecutive sputum samples sent for acid fast bacilli staining
b) Three consecutive sputum samples sent for TB culture
c) Transbronchial biopsy
d) Quantiferon Gold Assay
e) Tuberculin Skin Test

A

Answer B

Mycobacterial culture, has the highest sensitivity for diagnosing and confirming active TB, requires 2 to 6 weeks for interpretation

How well did you know this?
1
Not at all
2
3
4
5
Perfectly
44
Q

RACP 2017a Q18

A 72-year-old female has a two-week history of episodes of shooting pain affecting her cheek and
jaw. The episodes last 30 seconds and she had dozens per day. They are brought on by chewing or
talking. She has a normal neurological examination. What is the most likely diagnosis?

a) Giant Cell Arteritis
b) Hemifacial Spasms
c) Migraine
d) Transient Ischaemic Attack
e) Trigeminal Neuralgia

A

Answer ? A

Jaw claudication caused by GCA, ischemia to the muscles of mastication

How well did you know this?
1
Not at all
2
3
4
5
Perfectly
45
Q

RACP 2017a Q20

A 40-year-old lady presents with fatigue, arthralgias, rash to her lower limbs, abdominal pain, fevers,
haematuria. CT angiography demonstrates micro-aneurysms to her renal and mesenteric arteries.
Which of the following auto-antibodies will likely be positive?

a) c-ANCA
b) Rheumatoid Factor
c) All antibodies are likely to be negative
d) p-ANCA
e) ds-DNA

A

Answer C

Diagnosis is polyarteritis nodosa, not associated with any specific antibody

How well did you know this?
1
Not at all
2
3
4
5
Perfectly
46
Q

RACP 2017a Q45

Calcinosis cutis is most commonly found in what condition?
a) Systemic Sclerosis
b) Systemic Lupus Erythematosus
c) Sarcoidosis
d) Bechet’s Disease
e) Sjogren’s Syndrome

A

Answer A
Dermatomyositis and systemic sclerosis were the most common ACTDs associated with calcinosis cutis

How well did you know this?
1
Not at all
2
3
4
5
Perfectly
47
Q

RACP 2017a Q63

A 79-year-old woman presents with three-month history of dull right thigh pain. She does not recall any injuries to her leg or hip. Her past medical history is significant for COPD, GORD and
Osteoporosis.
A radiograph of her hip is presented below

What is the most likely cause of her thigh pain?
a) Osteoarthritis
b) Paget’s disease involving the femur
c) Osteophyte formation
d) Atypical femoral fracture
e) Avascular Necrosis

A

Answer D Atypical femoral fracture

Rare complication of bisphosphonate therapy
Initial symptoms are thigh pain

X ray showing cortical thickening with fracture line

How well did you know this?
1
Not at all
2
3
4
5
Perfectly
48
Q

RACP 2017 Q80

A 75-year-old woman with back pain has a DEXA scan to investigate for Osteoporosis. The results are shown below:
T-score
Lumbar + 1
Radial - 2.6
Right Hip - 2.5
What is the most correct interpretation of the discordant T-scores?

a) Corticosteroid-induced Osteoporosis
b) Osteoarthritis of the lumbar spine
c) Osteomalacia
d) Osteoporosis
e) Primary Hyperparathyroidism

A

Answer B OA of the lumbar spine

How well did you know this?
1
Not at all
2
3
4
5
Perfectly
49
Q

RACP 2017 Q73

The ultrasound image presented below is that of the femoral condyle in the longitudinal plane. The
area of echogenicity in the cartilage is known as a double contour.
This is imaging finding is pathognomonic for which of the following conditions?
a) Chrondocalcinosis
b) Gout
c) Systemic Lupus Erythematosus
d) Psoriatic Arthritis
e) Rheumatoid Arthritis

A

Answer B

Double contour sign is a hyperechoic band over the superficial margin of the anechoic hyaline cartilage, which is highly specific for gout. In chondrocalcinosis, calcium deposits appear as hyperechoic foci within the intermediate layer of the cartilage.

How well did you know this?
1
Not at all
2
3
4
5
Perfectly
50
Q

RACP 2017 Q 99

a) Psoriatic Arthritis
b) Rheumatoid Arthritis
c) Haemochromatosis
d) Systemic Lupus Erythematosus
e) Ankylosing Spondylitis
f) Osteoarthritis
g) Polymyalgia Rheumatica
h) Sjogren’s Syndrome
i) Gout

Which of the above medical conditions is most likely in a patient who presents with:

Erosive arthritis of the 2nd and 3rd metacarpophalygeal joints.

A

Answer C Haemochromatosis

Squared-off bone ends and hook-like osteophytes in the MCP joints, particularly if these changes are located in the second and third MCP joints. Such changes are especially common in hemochromatosis [35] and hemochromatosis-associated CPPD disease

How well did you know this?
1
Not at all
2
3
4
5
Perfectly
51
Q

RACP 2017 Q100

a) Psoriatic Arthritis
b) Rheumatoid Arthritis
c) Haemochromatosis
d) Systemic Lupus Erythematosus
e) Ankylosing Spondylitis
f) Osteoarthritis
g) Polymyalgia Rheumatica
h) Sjogren’s Syndrome
i) Gout

Which of the above medical conditions is most likely in a

Reducible non-erosive inflammatory polyarthritis of the small joints of the hands

A

Answer D SLE

The arthritis of SLE is typically nonerosive and is generally different from that of rheumatoid arthritis (RA). The arthritis and arthralgias of SLE tend to be migratory; symptoms in a particular joint may be gone within 24 hours but can often last longer. Although any joint can be involved, the arthritis is usually symmetric and polyarticular, with a predilection for the knees, carpal joints, and joints of the fingers, especially the proximal interphalangeal (PIP) joint. The ankles, elbows, shoulders, and hips are less frequently involved.

How well did you know this?
1
Not at all
2
3
4
5
Perfectly
52
Q

RACP 2017b Q10

The Rheumatoid Factor antibody is directed against which of the following?
a) Nuclear antigens
b) Cyclic citrullinated peptides
c) Deoxy-ribonucleic acid
d) Fc portion of IgG
e) Nuclear cytoplasmic antigens

A

Answer D Fc portion of IgG

How well did you know this?
1
Not at all
2
3
4
5
Perfectly
53
Q

RACP 2017b Q43

72-year-old female presents with posterior ankle pain. There is no joint effusion on ultrasound to
aspirate. Dual energy CT shows sodium deposition in the Achilles tendon. What is the most likely
cause of ankle pain?
Below is an image of a dual-energy CT of the foot of the patient. The green areas represent sodium and the purple areas represent calcium. What is the most likely cause of her ankle pain?
a) Degenerative changes
b) Osteoarthritis
c) Gout
d) Rheumatoid Arthritis
e) Psoriatic Arthritis

A

Answer C Gout

Monosodium urate deposition seen in Achilles and also other common gout sites (first MTP/ podagra).

How well did you know this?
1
Not at all
2
3
4
5
Perfectly
54
Q

RACP 2017b Q50

Which of the following describes the target of the monoclonal antibody Denosumab?
a) Osteoblasts
b) Osteoclasts
c) RANK-Ligand
d) Estrogen receptors
e) PTH Receptor

A

Answer C RANK Ligand

Denusomab targets and binds RANK Ligand, inhibiting osteoclast formation, function, and survival. RANKL is a protein that acts as the primary signal to promote bone removal/resorption

How well did you know this?
1
Not at all
2
3
4
5
Perfectly
55
Q

RACP 2017b Q67

Options
a) Systemic Lupus Erythematosus
b) Polyarteritis Nodosa
c) Bechet’s Disease
d) Reactive Arthritis
e) Ankylosing Spondylitis
f) Lofgren Syndrome
g) Osteoarthritis
h) Secondary Syphilis

Which of the above conditions best fits the following clinical vignettes?

A Turkish man with conjunctivitis, arthritis, thrombophlebitis, oral and genital ulcers.

A

Answer C Bechets disease

How well did you know this?
1
Not at all
2
3
4
5
Perfectly
56
Q

RACP 2017b Q68

Options
a) Systemic Lupus Erythematosus
b) Polyarteritis Nodosa
c) Bechet’s Disease
d) Reactive Arthritis
e) Ankylosing Spondylitis
f) Lofgren Syndrome
g) Osteoarthritis
h) Secondary Syphilis

Which of the above conditions best fits the following clinical vignettes?

A 65-year-old woman presents with arthropathy involving several of her distal interphalangeal joints. She had normal ESR and CRP. The CCP is mildly elevated.

A

Answer G OA

How well did you know this?
1
Not at all
2
3
4
5
Perfectly
57
Q

RACP 2016a Q5

Question 5
Defects in apoptosis resulting in delayed elimination of self-proteins is thought to be involved in the pathogenesis of which of the following diseases?
A. Immune thrombocytopenia purpura
B. Multiple Sclerosis
C. SLE
D. Type 1 diabetes mellitus
E. Ulcerative Colitis

A
How well did you know this?
1
Not at all
2
3
4
5
Perfectly
58
Q

RACP 2016a Q50

Question 50
A 42 year old female presents with arthralgia and exertional dyspnoea. She reports episodes of Raynaud’s phenomenon has noticed weakness in both the upper and lower limbs. HRCT of the lungs shows evidence of ILD in
a NSIP pattern.
CK = 1500
ANA = 1: 1600
ENAs = Jo1 positive
RF = 30 (<20)
CCP = negative
What is the most likely diagnosis?
A) Anti-synthetase syndrome
B) SLE
C) RA
D) Polymyositis
E) Sjogren’s syndrome

A

Answer A
Antisynthetase syndrome

Antibodies to aminoacyl tRNA - anti-Jo-1, PL-7, PL-12 common
Clinical features depend on antibodies involved
Higher risk and greater severity of ILD
Other features – mechanics hand, Raynaud’s phenomenon and arthritis

How well did you know this?
1
Not at all
2
3
4
5
Perfectly
59
Q

RACP 2016a Q51

Question 51
An elderly Caucasian man presents with a long sand history of back pain and
stiffness. An x-ray is performed
The most likely diagnosis is
A) DISH
B) Hyperparathyroidism
C) Osteoarthritis
D) Psoriatic arthritis
E) Ankylosing spondylitis

A
How well did you know this?
1
Not at all
2
3
4
5
Perfectly
60
Q

RACP 2016a Q63

A young man presents with ankle swelling bilaterally and a painful red eye. He is also noted to have painful erythematous nodules on his lower limbs. He denies a history of unprotected intercourse. What is to most likely diagnosis?
A) Rheumatoid arthritis
B) Reactive arthritis
C) Sarcoidosis
D) Small vessel vasculitis
E) Psoriatic arthritis

A
How well did you know this?
1
Not at all
2
3
4
5
Perfectly
61
Q

RACP 2016a Q71

Question 71
Eliciting hip joint pathology is best achieved by which of the following
A) Extension, external rotation, abduction
B) Extension internal rotation, abduction
C) Extension, internal rotation, adduction
D) Flexion, external rotation, abduction
E) Flexion, internal rotation, abduction

A
How well did you know this?
1
Not at all
2
3
4
5
Perfectly
62
Q

RACP 2016a Q89

Question 89
34 year oldmale with a 4year history oflower back pain and morning stiffness. Pelvic X-ray reveals bilateral
grade 3 sacroilitis (film not shown). ESR = 34.
Apart from commencinghim on anexercise program what would be your next step?
A. Etanacept
B. Methotrexate
C. NSAIDs
D. Paracetamol
E. Prednisone

A
How well did you know this?
1
Not at all
2
3
4
5
Perfectly
63
Q

RACP 2016b Q13

Question 13
Which of the following drugs can precipitate a scleroderma renal crisis?
A. Prednisone
B. Hydroxychloroquine
C. Irbesartan
D. Adenosine
E. Amlodipine

A
How well did you know this?
1
Not at all
2
3
4
5
Perfectly
64
Q

RACP 2016b Q23

Question 23
54 year old man presents with bilateral proximal muscle weakness. His CK is elevated at 3400 U/L (<250 U/L).
Muscle biopsy demonstrates muscle necrosis without inflammation.
What is the most likely diagnosis?
A. Dermatomyositis
B. Mitochondrial myopathy
C. Necrotising autoimmune myopathy
D. Polymyositis
E. Viral myositis

A

Answer C Necrotising myopathy

Rapidly progressing proximal muscle weakness
Myalgia
Limited extra-muscular features and systemic involvement
Significantly elevated CK
Biopsy shows myonecrosis without inflammation

How well did you know this?
1
Not at all
2
3
4
5
Perfectly
65
Q

RACP 2016b Q26

Question 26
Which of the following antibodies is related with ILD and scleroderma
A. Anti smith
B. Anti-topoisomerase I
C. Anti RNA polymerase III
D. Anti centromere
E. Ds-DNA

A
How well did you know this?
1
Not at all
2
3
4
5
Perfectly
66
Q

RACP 2016b Q32

A 68 year old woman presents to your office with a history of long standing pain in her feet. A radiograph of her
feet shows lateral deviation of both feet with erosions in the proximal and distal intertarsal joints.
What is the most likely diagnosis
A) Charcot’s Arthropathy
B) Rheumatoid Arthritis
C) Osteoarthritis
D) Systemic Lupus Erythamatosis
E) Psoriatic Athritis

A
How well did you know this?
1
Not at all
2
3
4
5
Perfectly
67
Q

RACP 2016b Q53

Question 53
When given orally what is the comparative dose of prednisone to hydrocortisone 100mg?
A) 4mg
B) 10mg
C) 25mg
D) 40mg
E) 100mg

A
How well did you know this?
1
Not at all
2
3
4
5
Perfectly
68
Q

RACP 2016b Q62

Question 62
What is a contraindication to treatment with an anti-TNF alpha agent?
A) History of a demyelinating condition
B) Pancreatitis
C) Liver failure
D) Kidney failure
E) Previously treated TB

A
How well did you know this?
1
Not at all
2
3
4
5
Perfectly
69
Q

RACP 2015a Q23

Question 23
What is the mechanism of action of Teriparatide?
A. Increases osteoblast action
B. Reduces osteoclast action
C. Increases gastrointestinal absorption of calcium
D. Increases hydroxyl-vitamin D
E. Increases calcium secretion

A
How well did you know this?
1
Not at all
2
3
4
5
Perfectly
70
Q

RACP 2015a Q25

Question 25
What is most associated with rheumatoid arthritis severity?
A. Rheumatoid factor
B. Age
C. Smoking
D. Obesity
E. Rhubarb

A
How well did you know this?
1
Not at all
2
3
4
5
Perfectly
71
Q

RACP 2015a Q41

Question 41
A man has anterior uveitis treated with topical corticosteroids. Which is the most important
musculoskeletal abnormality to examine?
A. Shoulders
B. Lumbar spine
C. Achilles tendon
D. Hands
E. Knees

A
How well did you know this?
1
Not at all
2
3
4
5
Perfectly
72
Q

RACP 2015a Q43

Question 43
Which complication leads to greatest mortality in scleroderma patients?
A. GI disease
B. Cardiac disease
C. Renal disease
D. Drug toxicity
E. Respiratory disease

A
How well did you know this?
1
Not at all
2
3
4
5
Perfectly
73
Q

RACP 2015a Q51

Question 51
A 46 year old lady presents to the clinic, complaining of hand numbness. She is known to have seropositive rheumatoid arthritis and is treated with methotrexate, leflunomide and
prednisone. On examination, she is found to have sensory loss over the palmar aspect of
the thumb, index, middle and radial surface of the ring finger. She also has weakness of wrist flexion.
The most likely cause of her symptoms is:
A. Carpal tunnel syndrome
B. C6 radiculopathy
C. Pronator syndrome
D. Drug related neuropathy
E. Vasculitis

A
How well did you know this?
1
Not at all
2
3
4
5
Perfectly
74
Q

RACP 2015a Q92

Question 92
Which antibody is most specific for SLE?
A. Anti-CCP
B. Anti-Smith
C. Anti-dsDNA
D. Anti-Ro
E. Anticardiolipin IgG

A
How well did you know this?
1
Not at all
2
3
4
5
Perfectly
75
Q

RACP 2015b Q69

Question 92
Which antibody is most specific for SLE?
A. Anti-CCP
B. Anti-Smith
C. Anti-dsDNA
D. Anti-Ro
E. Anticardiolipin IgG

A
How well did you know this?
1
Not at all
2
3
4
5
Perfectly
76
Q

RACP 2015b Q70

Extended Matching Questions: Question 69-70
A. Scl70
B. Anti centromere
C. ANCA
D. anti-CCP
E. anti-SSA
F. anti-SSB
G. anti dsDNA

Question 70:
32 year old female, 12 weeks pregnant, has sicca symptoms.
Which antibody predicts development of neonatal heart-block?

A
How well did you know this?
1
Not at all
2
3
4
5
Perfectly
77
Q

RACP 2015

An older man presents with gout. He is on frusemide and hydrochlorothiazide. You
commence him on allopurinol.
What is the best endpoint to assess the efficacy of his allopurinol dose?
A. Urate level
B. Swollen joint count
C. CRP
D. Rhubarb
E. Rhubarb

A

Answer A Urate/Uric acid level

Urate lowering therapy is commenced with treat to target uric acid level:

< 0.36mmol/L for non-severe gout
<0.30 mmol/L for severe/ tophaceous gout

How well did you know this?
1
Not at all
2
3
4
5
Perfectly
78
Q

RACP 2014a Q28

79 year old lady presents to hospital with an acute sore, swollen R shoulder. She has a background history of rheumatoid arthritis treated with adalimumab/steroids. She is febrile to 39.5C and has tenderness, swelling and limited range of movement with the right shoulder. Joint aspirate reveals a cell count of 120 000 (90% neutrophils) and negative gram stain. What
is it most likely to be
a. RA
b. Gout
c. OA
d. Septic arthritis

A

Answer D

Reasons pointing to septic arthritis:
- High WBC count with high neutrophil percentage (non-infectious inflammation have WBc counts upto 50,000 max, non- inflammatory should be < 2000 WBC)
- High grade fevers
Immunosuppression for RA treatment
-Gram stains are negative in about 50% of cases of bacterial arthritis, but (in the absence of antibiotics) cultures are positive in about 90%

How well did you know this?
1
Not at all
2
3
4
5
Perfectly
79
Q

RACP 2014b Q5

Question 5:
A 76 year old female is evaluated for osteoporosis. She is currently on vitamin D and calcium medications. She states that she drinks milk regularly and exercises 20 minutes a day. She completed menopause at the age of 46 and never had any HRT. Her UEC, CMP and vitamin D are all normal. Her DEXA scan results are shown below:
T score:
Lumbar spine -1.5
Hip -1.2
What is the management plan?
1. Continue calcium and vitamin D
2. Commence HRT
3. Commence strontium
4. Commence bisphosphonate
5. Commence raloxifene

A
How well did you know this?
1
Not at all
2
3
4
5
Perfectly
80
Q

RACP 2014b Q7

Question 7
A 43 year old woman presents with painful dysaesthesias of both hands, as well as weakness of
dorsiflexion of the left foot. She also reports sicca symptoms and arthralgia. Results of some of her blood tests are outlined below:
ANA positive 1:2560
Anti-SSA and SSB antibodies positive
Rheumatoid factor positive
ANCA negative
Normal C3 and C4
What is the most likely underlying pathology?
a. Rheumatoid arthritis
b. Primary Sjogren’s syndrome
c. Mixed connective tissue disease
d. SLE
e. Cryoglobulinaemic vasculitis

A

Answer B Primary Sjogrens

How well did you know this?
1
Not at all
2
3
4
5
Perfectly
81
Q

RACP 2014b Q14

A 76 y.o female presents with difficulty forming a fist bilaterally. She has been having progressive difficulty with using a key over the last 12 months. She was wasting of the distal forearms and hands with associated weakness of the hands. She is ANA and ANCA –ve and has
a normal FBC, LFT and CK.
What is the likely diagnosis?
A. Hypothyroidism
B. Inclusion body myosotis
C. Polymyositis
D. SLE
E. Viral myopathy

A
How well did you know this?
1
Not at all
2
3
4
5
Perfectly
82
Q

RACP 2014b Q37

A 35 year old overweight woman (BMI 28) presents with 12 month history of bilateral shoulder
pain, dry eyes, face flushing after alcohol and urinary frequency.
Investigations showed ANA 1:160, ENA, RF negative. Full blood count, creatinine, liver function
tests and inflammatory markers were unremarkable.
What is the most likely diagnosis?
A. Fibromyalgia
B. Sjogrens
C. Systemic lupus erythematosis
D. Polymyositis
E. Polymyalgia rheumatica

A
How well did you know this?
1
Not at all
2
3
4
5
Perfectly
83
Q

RACP 2014 Q41

A 58 year old gentleman presents with a two week history of right shoulder pain, worse at night time. He has restricted ROM both to passive and active movement. His ESR, CRP
are normal and hisANA is negative. What is the most likely diagnosis?
A. Osteoarthritis
B. Rheumatoid arthritis
C. Gout
D. Adhesive Capsulitis
E. Septic arthritis

A

Answer D: Adhesive capsulitis

Clinical features:
Severe restriction of both active and passive range of motion of the glenohumeral joint in all planes, especially:
- External rotation
- Passive abduction (restricted to < 90°)

How well did you know this?
1
Not at all
2
3
4
5
Perfectly
84
Q

RACP 2013a Q2

Question 2
Which is the best clinical feature to differentiate ankylosing spondylitis from mechanical back pain?
A. Loss of lumbar lordosis
B. Morning stiffness
C. Pain radiating to thigh
D. Age of onset
E. Reduced lumbar movement

A

Answer B Morning stiffness

Ankylosing spondylitis causes inflammatory back pain characterised by:
- slow insidious onset
- nocturnal pain
- morning stiffness that improves with activity
- persisting of pain at rest

How well did you know this?
1
Not at all
2
3
4
5
Perfectly
85
Q

RACP 2013a Q12

Question 12
A 60 year old male presents with a background of left knee osteoarthritis presents with new onset left knee
and posterior lower leg pain. The pain is worse on dorsiflexion.
What is the most likely pathology?

A) Meniscal Tear
B) A ruptured baker's cyst
C) DVT
D) S1 radiculopathy
E) Achilles tendon rupture

A
How well did you know this?
1
Not at all
2
3
4
5
Perfectly
86
Q

RACP 2013a Q62

Question 62
A 76 year old woman suffers a vertebral fracture due to osteoporosis. She is treated with IV zolendronate.
In addition to reducing rate of refracture, what is the benefit of bisphosphonates?
A. Improvement in mobility
B. Decreased bone pain
C. Improved QOL
D. Decrease mortality
E. Improved muscle strength

A
How well did you know this?
1
Not at all
2
3
4
5
Perfectly
87
Q

RACP 2013b Q6

Question 6
A middle aged patient presents with headache, lethargy, lymphadenopathy and a purpuric rash on his feet. Bloods demonstrate
Cryocrit 75%
EPG paraprotein 2g/L
IEPG monoclonal IgM
C4 low limit of normal
RF elevated
What is the most likely diagnosis?
a) Hepatitis C infection
b) Rheumatoid arthritis
c) Sjogren syndrome
d) Waldenstrom’s macroglobulinemia
e) Systemic lupus erythematous

A
How well did you know this?
1
Not at all
2
3
4
5
Perfectly
88
Q

RACP 2013b Q15

Question 15
A 30 year old woman presents with bilateral proximal muscle weakness. She is noted to have blanching of
the fingers, especially in cold weather.
Anti-Jo: positive
ANA positive
CK 2000s
ESR 60
EMG: myopathic changes

The presence of which of the following conveys the worst prognosis?
A) Dysphagia
B) Skin rash
C) CK > 2000 ( or >10xULN)
D) ILD
E) Raynaud’s Phenomenon

A
How well did you know this?
1
Not at all
2
3
4
5
Perfectly
89
Q

RACP 2013b Q39

Question 39
A 60 year old diagnosed with polymyositis which has been well controlled for the last six months with prednisolone 30mg daily and methotrexate 15mg daily. For the last two weeks he has reported worsening proximal weakness and trouble walking up the stairs. Creatine phosphokinase and liver function tests are normal.
What would be the most appropriate next step in management:
A) Commence cyclosporin
B) Increase methotrexate dose
C) Muscle biopsy
D) Physiotherapy
E) Decrease prednisolone dose

A
How well did you know this?
1
Not at all
2
3
4
5
Perfectly
90
Q

RACP 2013b Q51

Extended match questions
Options:
a) Anti CCP
b) Anti Sm
c) Anti ANCA
d) Anti Scl 70
e) Anti DNA
f) Rh factor
g) Anti-centromere
h) Anti-Jo
i) Anti-RNP

What is the most diagnostic test for the conditions below?

Question 51
41 year old woman presents with tightening of fingers, mild difficulty swallowing, and mild shortness of
breath on exertion. She takes somac for reflux. On examination there is tightening of skin in her fingers,
however the rest of the skin is normal. Her joints are not inflamed. The rest of her examination was normal
(including chest examination). Her CXR is also normal. There is mild decrease in DLCO on lung function tests

A
How well did you know this?
1
Not at all
2
3
4
5
Perfectly
91
Q

RACP 2013b Q52

Extended match questions
Options:
a) Anti CCP
b) Anti Sm
c) Anti ANCA
d) Anti Scl 70
e) Anti DNA
f) Rh factor
g) Anti-centromere
h) Anti-Jo
i) Anti-RNP

What is the most diagnostic test for the conditions below?

Question 52
A 30 year old woman presents with fatigue, weight loss and swelling of the small joints of her hands, worst
in the morning, improves with exercise, present for several months. There was no swelling or erythema on
examination, but tenderness on mobilising the small joints of her hands. Her urinalysis is clear and her
respiratory function tests are normal. She also has a rash on her trunk and arms. What is the most useful
test to establish her diagnosis?

A
How well did you know this?
1
Not at all
2
3
4
5
Perfectly
92
Q

RACP 2013b Q55

30F presents with polyarthralgia and fatigue. Which of the following conditions is a negative ANA most
useful for ruling out?
A. Fibromyalgia
B. Adult onset Still's disease
C. Systemic lupus erythematosus
D. Scleroderma
E. Rheumatoid arthritis

A
How well did you know this?
1
Not at all
2
3
4
5
Perfectly
93
Q

RACP 2013b Q67

A 66yo female presents with hand pain. This is an x-ray. What is the diagnosis?

A) Osteoarthritis
B) Rheumatoid arthritis
C) Chronic tophaceous gout
D) Psoriatic arthritis
E) Scleroderma

A
How well did you know this?
1
Not at all
2
3
4
5
Perfectly
94
Q

RACP 2013b Q82

Question 82
With respect to osteoarthritis of the knee, which of the following is most effective in long-term
management of pain?
A. Arthroscopic lavage
B. Glucosamine
C. Muscle strengthening exercises
D. Paracetamol
E. NSAIDs

A
How well did you know this?
1
Not at all
2
3
4
5
Perfectly
95
Q

RACP 2012a Q35

QUESTION 35
Which of the following clinical features is most useful in differentiating between ankylosing spondylitis and mechanical lower back pain?
A. Age
B. Early morning back stiffness
C. Loss of lumbar lordosis
D. Loss of lumbar flexion
E. Pain radiating from the back to posterior thighs

A

Answer ?B

How well did you know this?
1
Not at all
2
3
4
5
Perfectly
96
Q

RACP 2012b Q48

QUESTION 48
Corticosteroid induced osteoporosis, as compared to postmenopausal
osteoporosis, is more likely to be associated with which of the following?
A. Asymptomatic vertebral fractures
B. Fractures at a higher BMD
C. Decreased response to calcium
D. Decreased response to bisphosphonates
E. Sub trochanteric fractures

A
How well did you know this?
1
Not at all
2
3
4
5
Perfectly
97
Q

RACP 2012b Q2

Which of the following risk factor is the best predictor of a poor recovery from acute
lower back pain?
A. Obesity
B. Job dissatisfaction
C. Age >60
D. Sedentary occupation
E. Poor physical fitness

A
98
Q

RACP 2012b Q32

A lady presents with polyarthralgia, fever and rashes on her hands. An example of
her rash is below. What is the condition likely to be?
A. SLE
B. MCTD
C. Scleroderma
D. Cryoglobulinemia
E. RA

A
99
Q

RACP 2012b Q36

A 45 year old lady has a history of bloody diarrhoea two weeks previously which was treated with Amoxil. She now presents with lethargy and a petechial rash on her
lower limbs
Creatinine 670
Hb 81
WCC 3.4
Platelets 120
C3 and C4 normal
The blood film showed a left shift with schistocytes.
What is the most likely diagnosis?
A. HUS
B. Henoch shoenlein
C. Vasculitis
D. SLE
E. Cryoglobulinaemia

A
100
Q

RACP 2012b Q47

A 54 year old man has a history of gout and hypertension and is on long term allopurinol. The patient is then started on frusemide but develops acute gout in the right ankle.
What is most appropriate management?
A. Stop allopurinol
B. Stop frusemide
C. Paracetamol
D. Naproxen
E. Prednisone

A
101
Q

RACP 2012b Q52

A 30 year old female presents with polyarthralgia and fatigue. A negative ANA is most useful in excluding which of the following
A. Adult onset stills disease
B. Fibromyalgia
C. Rheumatoid arthritis
D. Scleroderma
E. SLE

A
102
Q

RACP 2012b Q78

A lady with rheumatoid arthritis presents with the following signs when attempting to
extend her fingers:

The most likely diagnosis is:
A. Rupture of extensor tendons
B. Dupytrens contracture
C. Arthritis of MCP joints
D. Radial nerve palsy
E. Subluxation of MCP joints

A
103
Q

RACP 2012b Q88

A patient presents with painful hands with the appearance shown below:

What type of arthritis commonly presents with this finding?
A. Enteropathic arthritis
B. Psoriatic arthritis
C. Reactive arthritis
D. Rheumatoid arthritis
E. SLE

A
104
Q

RACP 2011a Q1

Which of the following joints is most commonly involved in pseudogout (acute calcium pyrophosphate deposition disease)?
A. Ankle.
B. Elbow.
C. First metatarsophalangeal joint.
D. Knee.
E. Wrist.

A

Answer D Knee

105
Q

RACP 2011a Q18
QUESTION 18
What is the most common initial clinical feature of scleroderma?
A. Diffuse swelling of the hands.
B. Dysphagia.
C. Dyspnoea.
D. Pruritis.
E. Raynaud phenomenon.

A
106
Q

RACP 2011a Q16

75 year old man with a history of back pain had a DEXA scan with the
following T scores:

Lumbar spine +0.5
Hip -2.5
Radius -2.5

What is the best explanation for the discordant T score of the lumbar
spine?

A. Glucocorticoid induced osteoporosis
B. Osteomalacia
C. Osteoarthritis of the lumbar spine
D. Osteopetrosis
E. Primary hyperparathyroidism

A
107
Q

RACP 2011a Q18

Which of these features has the highest likelihood ratio for giant cell
arteritis (GCA)?

A. Headache
B. Diplopia
C. Jaw claudication
D. Joint synovitis
E. Proximal myalgia

A
108
Q

RACP 2011a Q61

Question 61
Which of the following will most increase the risk of septic arthritis with
rheumatoid arthritis?
A. Corticosteroids
B. Etanercept
C. Lefunomide
D. Methotrexate
E. Sulphasalazine

A
109
Q

RACP 2011b Q13

In a patient with Rheumatoid Arthritis, what is the most appropriate time to
commence DMARDs?
A. If Rh factor is +ve
B. at the time of diagnosis
C. when CRP is > 15mg/L
D. when > 4 joints are involved
E. when patient develops erosions

A
110
Q

RACP 2011b Q19

Question 19
Mechanism of steroid induced osteoporosis is:
A. Increased apoptosis of osteoblasts
B. Increased intestinal Ca2+ absorption
C. Increase parathyroid hormone
D. Decreased production of osteoprotegerin
E. Decreased renal tubular Ca secretion

A
111
Q

RACP 2011b Q29

A 54 year old male presents with an 18 month history of pain and swelling in the
distal interphalangeal joint of his right index finger. An X-ray is shown:

The most likely diagnosis is:
A. Calcium pyrophosphate deposition disease
B. Gout
C. Osteoarthritis
D. Psoriatic arthritis
E. Septic arthritis

A
112
Q

RACP 2011b Q44

Question 44
The drug most likely to inhibit CaCO3 absorption is:
A. Omeprazole
B. Cholesteramine
C. Alendronate
D. Rifampacin
E. Thiazide

A
113
Q

RACP 2011b Q48

A male patient with a history of intravenous drug use presents with a rash, myalgias, arthralgias. He is afebrile. He is positive to hepatitis C antibody and rheumatoid factor. He C4 complement level is reduced with normal C3 levels. His ANA is positive
at 1:160 speckled. His rash is shown below:

What is the most likely diagnosis?
A. Cryoglobulinaemic vasculitis
B. Henosch-schonlein purpura
C. Rheumatoid vasculitis
D. Subacute bacterial endocarditis
E. Meningococcal disease

A
114
Q

RACP 2011b Q51

Question 51
A 65 year old male with longstanding asthma presents with cough, dypsnoea and a
right 6 th cranial nerve palsy. A CT scan shows diffuse and patchy ground glass
changes.

His blood tests are shown:
Hb 109 [130-180]
WCC 14 [4-11]
Plt 480 [150-400]
ANCA positive

RPA2011_Paper B Page 25/17
Anti-myeloperoxidase (MPO) 99 [<5]
ESR 100 [0-5]
CRP 30 [0-5]
Anti-dsDNA 5 [0-6]
ANA 1:160 speckled
Urinalysis trace blood, no protein
The most likely diagnosis is:
A. Allergic bronchopulmonary aspergillosis (ABPA)
B. Churg-Strauss syndrome
C. Primary hypereosinophilic syndrome
D. Systemic lupus erythematosus
E. Wegener’s granulomatosis

A
115
Q

RACP 2011b Q57

A 76 year old man presents with low back pain limiting his walking to 20 minutes on a
background of osteoarthritis. There are no neurological signs. A CT performed
demonstrates disc bulges at L3/4 and L4/5 with ligamentum flavum hypertrophy, facet
joint disease and osteophytes encroaching on to the neural root exit foramina and
spinal canal. Apart from regular paracetamol, what is the most appropriate treatment?
A. Back and abdominal muscle strengthening exercises
B. Epidural steroid injection
C. Lumbar corset
D. NSAIDs
E. Surgical decompression

A
116
Q

RACP 2011b Q66

A 30 year old man presents with pain and swelling at his heel as shown below.

The most likely cause is:
A. Achilles tendonitis
B. Achilles enthesopathy
C. Acute gout
D. Achilles tendon rupture
E. Posterior ankle impingement

A
117
Q

RACP 2011b Q92

A 40 year old man with a 20 year history oftype one diabetesmellitus presents with
difficulty flexing and extending his fingers. There is thickened skin over the dorsum
of the hands with palpable thickened tendon sheaths. There is no digital pitting or
loss of distal finger subcutaneous tissue. What is the most likely diagnosis?

RPA2011_Paper B Page 43/17
A. Hypothyroidism
B. Limited scleroderma
C. Haemachromatosis
D. Diabetic cheiroarthropathy
E. Amyloidosis

A
118
Q

RACP 2011b Q97

A 27 year old lady with SLE presents with recurrent miscarriage. Which antibody
predicts further miscarriages
A. Anti-SSa
B. Anti-histone
C. Anti-cardiolipin
D. Anti-dsDNA
E. Anti-Smith

A
119
Q

RACP 2010a Q11

Question 11
The Trendelenberg sign is elicited in patients with hip OA, and is positive when opposite side of the
pelvis tips down when standing on the affected leg. It indicates weakness of which of the following
muscles:
A. Gluteus maximus
B. Gluteus medius
C. Iliopsoas
D. Piriformis
E. Tensor Fascia Lata

A
120
Q

RACP 2010a Q42

Question 42
Which rheumatologic disorder primarily involves the enthesis?
A. OA
B. RA
C. Pseudogout
D. Reactive arthritis
E. Ankylosing spondylitis

A

Answer : ? B

121
Q

RACP 2010a Q62

Question 62
What is the most common mechanism for primary hyperuricaemia?
A. Increased gastrointestinal absorption of uric acid
B. Increased cell turnover
C. Inherited defects in purine synthesis
D. Inherited defects in adenosine triphosphate (ATP) metabolism
E. Reduced uric acid urinary excretion

A
122
Q

RACP 2010b Q11

A 23 yo male presents with 1 week of arthralgias and 2 days of arthritis of the left wrist and right knee. On examination there is tenosynovitis of the extensor tendons of the left forearm, and a
pustular rash on the palm of his left hand. The most likely diagnosis is:
A. Ankylosing Spondylitis
B. Enteropathic arthritis
C. Gonococcal arthritis
D. Psoriatic arthritis
E. Reactive arthritis

A
123
Q

RACP 2010b Q16

A 34 year old man presents with acute low back pain. There is no previous history of back pain.
Examination revealed loss of lumbar lordosis and painful back movements. Lower limb neurological
exam was normal.
After adequate analgesia, what is the most appropriate intervention?
A. Acupuncture.
B. Advise to stay active.
C. Bed rest.
D. Manipulation.
E. Traction.

A
124
Q

RACP 2010b Q31

A 60 yo man presents with acute gout of the right knee and ankle. 3 months ago he was started on allopurinol. His Creatinine is 130 mmol/L and his Uric acid 0.44 mmol/L. What is the most appropriate management?
A. colchicine 0.5mg tds
B. diclofenac
C. predniso(lo)ne 25mg daily
D. probenecid
E. stop the allopurinol

A
125
Q

RACP 2010b Q47

A 54-year-old man with an 18 month history of pain and swelling in the DIP joint of the right index finger has the following x-ray. What is the likely diagnosis? The XR is shown below.

This is the closest XR I could find to the actual exam. The main differences are: 1. The DIP instead of PIP was involved. 2. There was significant osteopenia without any significant evidence of punchout lesions. (X-ray shows a hand, with some sort of intraosseous abnormality which some interpreted as
a subchondral cyst but which others have suggested as a tophus)

A. Gout
B. Pseudogout
C. Osteoarthritis
D. Psoriatic arthritis
E. Septic arthritis

A
126
Q

RACP 2010b Q70

A 70 yo male with presents with several years history of dysphagia and progressive muscle
weakness. On examination he has wasting of his distal forearm muscles and left quadriceps. His reflexes are intact. What is the most likely cause of his symptoms?
A. Dystrophinopathies
B. Facioscapulohumeral muscular dystrophy
C. Inclusion body myositis
D. Limb girdle dystrophy
E. Polymyositis

A

Answer C IBM

Characterised by asymmetrical weakness with distal muscle involvement (as opposed to other inflamm myopathies that involve proximal mm)
IBM - finger flexors and quadriceps first affected

127
Q

RACP 2010b Q85

62 yo man with hx of gout presents with acute painful arthritis of his 1st right MTP joint. He has mild renal impairment Cr. 136. What is the best treatment option?
A. NSAID’s
B. Allopurinol
C. Colchicine
D. Probenecid
E. Corticosteroids

A

Answer E Corticosteroids

NSAIDs contraindicated in renal impairment
Colchicine - needs dose adjustment in renal impairment.

128
Q

RACP 2010b Q95

A 58 year old woman presents with bilateral hand pain that only occurs with use of her hands. The pain affects both carpo-metacarpal joints and several DIP joints on both hands. On examination the affected joints are tender with decreased range of motion, but are not swollen. Her CRP is normal, rheumatoid factor 34 [0-14] and anti-CCP antibodies are normal. What is the best way to manage this
patient?
A. methotrexate
B. naproxen
C. paracetamol
D. Prednisolone
E. infliximab
F. Resting hand splints

A
129
Q

RACP 2010b Q98

40 year old man with a 20 year history of T1DM presents with difficulty flexing and extending his fingers. There is thickened skin over the dorsum of the hands with palpable thickened tendon sheaths. There is no digital pitting or loss of distal finger subcutaneous tissue.
What is the most likely diagnosis?
A. hypothyroidism
B. limited scleroderma
C. haemachromatosis
D. diabetic cheiroarthropathy
E. osteoarthritis

A
130
Q

RACP 2009a Q11

Pain during which of the following is most characteristic of osteoarthritis of the patello-femoral
joint?
a. Walking upstairs
b. Walking
c. Jogging
d. Standing up from chair
e. Standing

A
131
Q

RACP 2009a Question 20
OA of the hip most likely presents with pain in
a) Groin
b) Lateral thigh
c) Medial thigh
d) Knee
e) Buttock

A
132
Q

RACP 2009a Question 37
Which joint is most likely to be involved in pseudogout?
A) Ankle
B) Knee
C) 1st metacarpophalangeal
D) 1st carpometacarpal
E) Wrist

A

Answer B Knee

133
Q

RACP 2009a Question 51
Osteonecrosis of the jaw is a rare disorder of bisphosphanates. What is the greatest
risk factor for ONJ?
a) Advanced age
b) Mode of delivery ñ IV vs PO
c) Total exposure dose
d) Lack of Vit D and calcium co supplementation

A
134
Q

RACP 2009a Question 52
Apart from CRP and ESR, what is the best indicator of disease activity in rheumatoid
arthritis?
a. Anaemia
b. Leucocytosis
c. Rheumatoid factor
d. Thrombocytosis
e. Hypoalbuminaemia

A
135
Q

RACP 2009b Question 19
A 58 year old man history of psoriasis admitted to emergency having a swollen right knee after a long walk. An Xray shows chondrocalcinosis and loss of medial
cartilage (the X-Ray is not given in the exam).
Aspirate shows:
Clear and straw coloured
WCC 900
90% monocytes
Occasional gout crystals
Whats the diagnosis?
A. gout
B. pseudogout
C. osteoarthritis
D. psoriatic arthritis
E. medial cartilage tear

A
136
Q

RACP 2009b Question 50
A 79 yr old male presents with history of progressive difficulty walking over several
years. Currently he is able to walk 200m on the flat. He describes the feeling as if his legs won’t carry him. The feeling is relieved by rest and he is able to walk a further
short distance. His walking distance is increased if he leans over his shopping trolley.
He also has history of osteoarthritis and hyperlipidemia and is on treatment with
naproxen and atorvastatin.
What is the most likely cause?
A) Bilateral knee osteoarthritis
B) Proximal myalgia
C) Peripheral vascular disease
D) Polyradiculopathy
E) Lumbar spinal stenosis
F) Polymyositis

A
137
Q

RACP 2009b Question 51
A 50 yr old female presents with history of photosensitive rash over sun exposed
areas. Skin biopsy is consistent with discoid lupus erythematosis. There is no systemic
involvement.
The following are the serological test results:
ANA 1:160 speckled
dsDNA neg
ENA neg

Question also shows a picture with a typical butterfly rash.
What is the most appropriate initial line of management?
A) Sun avoidance and sunscreen
B) Hydroxychloroquine
C) Topical corticosteroids
D) Low dose oral steroids
E) Dapsone

A
138
Q

RACP 2009 Q 67

A patient presents with palpable purpura of the lower legs and buttocks. Biopsy demonstrates a leukocytoclastic vasculitis. Immunofluorescence shows heavy deposition of IgA in vessels. There is no protein or blood in the urine. What is the next
investigation?
a. Renal biopsy
b. Urine sediment
c. Faecal occult blood test
d. Throat swab
e. ANCA

A
139
Q

RACP 2009b Question 78
Middle age man presents with history of arthralgia, arthritis, and proximal muscle weakness. Investigations shows Positive ANA, Negative ENA, Anti-Scl 70, Anti Jo-1,
Ro/SSA, La/SSB, Rheumatoid factor, and positive RNP. CK is 1200. What is the most
consistent diagnosis.
1) SLE
2) Mixed Connective tissue disease
3) Rheumatoid arthritis
4) Polymyositis

A
140
Q

RACP 2009b Question 82 (Repeat from 2008)
A 55 year old man with a history of gout presents with an acutely swollen
metarsophalangeal joint. His creatinine is 152 and his urate is 0.55. What is the most
appropriate treatment?
a. Prednislone
b. Colchicine
c. NSAIDS
d. Allopurinol

A
141
Q

RACP 2009b Question 87
A 63 year old man presents with a 2 year history of slowly progressive muscle
weakness. He is found to have proximal weakness of both the upper and lower limbs as well as distal weakness of the upper limbs. Reflexes and sensation are intact. CK is
elevated at 636. CBE and LFTs are normal. What is the most likely diagnosis?
a. polymyositis
b.dermatomyositis
c. inclusion body myositis
d. polymyalgia rheumatica
e. recurrent relapsing polyneuropathy

A
142
Q

RACP 2009b Question 99
A 63 year old mn has had arthritis of his hands for the past 12 years. He now
complains of pain his his feet while walking. An X ray of his hands is shown.
What is the most likely cause of this manís condition?
A. Rheumatoid arthritis
B. Psoriatic arthritis
C. Haemachromatosis
D. Reactive arthritis
E. Osteoarthritis

A
143
Q

RACP 2008a QUESTION 4
Where is pain most likely to be felt in a patient with osteoarthritis of the hip?
A. Groin.
B. Buttock.
C. Lateral thigh.
D. Knee.
E. Medial thigh.

A
144
Q

RACP 2008a QUESTION 10
In a patient with rheumatoid arthritis, which clinical feature is most strongly correlated with the risk of
future joint erosions?
A. Length of early morning stiffness.
B. Number of tender joints.
C. Number of swollen joints.
D. Severity of joint pain.
E. Amount of weight loss.

A
145
Q

RACP 2008b QUESTION 7
When aspirating a small effusion in the knee joint, where should the needle be inserted (see below)?

A. 1 cm medial to the junction of the upper and middle third of the patella.
B. 3 cm medial to the lower pole of the patella.
C. 1 cm medial to the mid point of the patella tendon.
D. 1 cm lateral to the lower pole of the patella.
E. 3 cm lateral to the junction of the upper and middle third of the patella.

A
146
Q

RACP 2008b QUESTION 29
A week after a flu like illness, a 22-year-old presents with a haemorrhagic rash on the lower extremities (as shown in the figure below), and generalised arthralgia. The serum creatinine is elevated (125 μmol/L, [<110 μmol/L]) and there is blood and protein in the urine.

The most likely finding on renal biopsy is:
A. glomerular microangiopathy (haemolytic-uraemic syndrome).
B. mesangiocapillary glomerulonephritis.
C. Henoch-Schönlein nephropathy.
D. antineutrophil cytoplasmic antibody (ANCA) positive vasculitis.
E. lupus nephropathy.

A
147
Q

RACP 2008b Q 44

A 43-year-old man with human immunodeficiency virus (HIV), first diagnosed 20 years previously,
presents with a gradual onset of right-sided hip pain. He is on tenofovir, emtracitabine and efavirenz. He has had no acquired immunodeficiency syndrome (AIDS) defining illnesses in the past. On examination he has evidence of facial lipoatrophy and there is limitation of the range of movement of
the right hip because of pain. All other joints are normal. There are no skin rashes. An X-ray of his hip is shown below.

Laboratory results show:
CD4 count 0.41 x 109/L [0.45 – 1.1 x 109/L]
HIV viral load undetectable
Full blood count (FBC) within normal limits
Uric acid 0.55 mmol/L [0.18 – 0.48 mmol/L]
Antinuclear antibodies not detected
Rheumatoid factor not detected

What is the most likely diagnosis?
A. Avascular necrosis.
B. Psoriatic arthritis.
C. Osteoarthritis.
D. Rheumatoid arthritis.
E. Gout.

A
148
Q

RACP 2008b QUESTION 47
The presence of which of the following is the best reason not to commence alendronate for a woman
with postmenopausal osteoporosis?
A. Oesophageal stricture.
B. Dental caries.
C. Hyperparathyroidism.
D. Serum creatinine 140 μmol/L.
E. Recent fracture.

A
149
Q

RACP 2008b QUESTION 48
A 30-year-old female presents with a painful swollen left calf. Duplex study demonstrates a proximal
deep vein thrombus extending into the ileofemoral venous system. The following laboratory results
are obtained:
Prothrombin time (PT) 16 sec [11 – 18 sec]
Activated partial thromboplastin time (APTT) 43 sec [25 – 36 sec]
Dilute Russell Viper venom test Lupus anticoagulant detected
Anticardiolipin IgG negative
Anticardiolipin IgM negative
Anti-beta 2 glycoprotein 1 moderate positive
Treatment with anticoagulation with heparin followed by warfarin therapy is commenced. Repeat testing at 12-weeks confirms a positive anti-beta 2 glycoprotein 1. What is the most appropriate duration of anticoagulation?
A. 3 months.
B. 6 months.
C. 12 months.
D. 24 months.
E. Lifelong.

A
150
Q

RACP 2008b QUESTION 54
In a 75-year-old woman presenting with a unilateral temporal headache, which of the following clinical
features is most specific for a diagnosis of temporal arteritis?
A. Temporal artery tenderness.
B. Blurred vision.
C. Jaw (masseter) claudication.
D. Pain and stiffness around the shoulders and hips.
E. Fever.

A
151
Q

RACP 2008b QUESTION 65
A 27-year-old man presents with an 18 month history of low back pain and stiffness, a swollen left knee and heel pain. Examination demonstrates reduced lumbar flexion and chest expansion. He is HLA-B27 positive and has a C-reactive protein (CRP) of 25 mg/L [<8 mg/L]. In addition to physiotherapy what is the most appropriate initial pharmacological therapy?
A. Prednisolone.
B. Naproxen.
C. Methotrexate.
D. Etanercept.
E. Salazopyrin.

A
152
Q

RACP 2008b QUESTION 93
A 54-year-old man with a previous history of gout presents with an acutely swollen painful left knee for 24 hours. On examination he is febrile (38.2oC) and has a large effusion of the left knee and limited range of motion due to pain. Aspiration of the joint reveals:
White cell count 50,000 cells/ml
Microscopy No crystals
Gram stain negative

The best initial treatment is:
A. flucloxacillin.
B. indomethacin.
C. prednisolone.
D. allopurinol.
E. colchicine.

A
153
Q

RACP 2007a QUESTION 18
Which of the following has the greatest effect on peak bone mass?
A. Pubertal age.
B. Childhood calcium intake.
C. Genetic factors.
D. Childhood physical activity.
E. Body weight.

A
154
Q

RACP 2007a QUESTION 21
A 74-year-old female presents with severe jaw pain following tooth extraction. The jaw is swollen and disfigured. The X-ray shows extensive changes around the site of extraction consistent with osteonecrosis. The medication most likely to cause this condition is:
A. ergocalciferol.
B. zolendronate.
C. calcitriol.
D. fluoride.
E. strontium.

A
155
Q

RACP 2007a QUESTION 31
Which of the following tests is most specific for a diagnosis of rheumatoid arthritis?
A. Anti-cyclic citrullinated peptide antibody.
B. Rheumatoid factor.
C. Antinuclear antibody.
D. Anti-Ro antibody.
E. Anti-neutrophil cytoplasmic antibody.

A
156
Q

RACP 2007a QUESTION 50
Which of the following is the most likely initial site of involvement in primary osteoarthritis of the knee?
A. Medial tibiofemoral cartilage.
B. Lateral tibiofemoral cartilage.
C. Patellofemoral cartilage.
D. Intercondylar cartilage.
E. All of the above.

A
157
Q

RACP 2007b QUESTION 5
A 79-year-old female living in her own home has had falls every week for the last six months and as a result has fractured her radius and a lumbar vertebra. She has a past history of hypertension, cataracts, depression and ischaemic heart disease and now mobilizes with a pick up frame. Which of
the following interventions will reduce her future risk of falling the most?

A. Vitamin D supplementation.
B. Withdrawal of antihypertensive medications.
C. Excision of cataracts.
D. Exercise programme.
E. Home visit and modifications by occupational therapist.

A
158
Q

RACP 2007b QUESTION 9
A 54-year-old female presents with a three month history of symmetrical polyarthralgia. The following laboratory results are obtained:
Antinuclear antibody (ANA) positive, titre: 1/160, pattern: speckled
titre 1/640, pattern: cytoplasmic
Antibodies to extractable nuclear antigens (ENAs) positive anti-SS-A (Ro)
Antibodies to ribosomal p protein positive
Anti-double stranded DNA antibody (anti-dsDNA) 4 IU/mL [0-5]
Anti-cyclic citrullinated peptide (anti-CCP) negative
Rheumatoid factor (RF) 45 IU/mL [0-20]
Which of the following is the most likely diagnosis?
A. Rheumatoid arthritis.
B. Systemic lupus erythematosus.
C. Sjogren’s syndrome.
D. Mixed connective tissue disease.
E. Systemic sclerosis.

A
159
Q

RACP 2007b QUESTION 14
A 65-year-old woman with longstanding rheumatoid arthritis and osteoporosis presents with a mid
thoracic crush fracture. She has a history of two previous crush fractures and has been taking alendronate 70mg weekly for three months. She has a normal serum calcium, phosphate, albumin and 25 hydroxy-vitamin D. She has taken prednisolone in the past.
Which of the following treatments is most appropriate for future fracture prevention?

A. Continue alendronate and add raloxifene.
B. Cease alendronate and commence intravenous pamidronate.
C. Continue alendronate.
D. Cease alendronate and commence strontium ranelate.
E. Cease alendronate and commence risedronate.

A
160
Q

RACP 2007b QUESTION 52
A 20-year-old female presents with a one month history of malaise, nausea, abdominal pain, itch and
polyarthralgia. She gives no history of recent use of prescribed or illicit drugs. Examination reveals
jaundice, multiple bruises, and tender hepatomegaly. Laboratory results are consistent with an acute
hepatocellular injury, with a polyclonal increase in IgG (immunoglobulin G) of 32g/L (6.1-15.5).
Serological tests for hepatitis A, B, and C viruses are negative. Histopathological findings on liver biopsy are of a periportal mixed mononuclear cell infiltrate of plasma cells, lymphocytes and eosinophils consistent with an autoimmune hepatitis.
Which of the following autoantibodies, if present, is most predictive of a poor clinical and biochemical response to therapy?
A. Anti-nuclear antibody.
B. Anti-smooth muscle antibody (F-actin–specific).
C. Anti-mitochondrial antibody.
D. Anti-liver-kidney microsomal antibody.
E. Atypical perinuclear anti-neutrophil cytoplasmic antibody.

A
161
Q

RACP 2007b QUESTION 69
A 49-year-old woman with a 25 year history of type 1 diabetes mellitus presents with progressive mid foot pain and swelling over six months. An X-ray of the affected foot is shown below.

The most likely diagnosis is:
A. Osteoarthrititis.
B. Neuropathic arthropathy.
C. Osteomyelitis.
D. Gout.
E. Rheumatoid arthritis.

A
162
Q

RACP 2007b QUESTION 99
A 72-year-old man presents with a right sided headache for two weeks. On examination he is tender over the temporal artery. His erythrocyte sedimentation rate (ESR) is 82 mm/hr [0-22].
Which is the most appropriate next investigation?
A. Ultrasound of temporal artery.
B. Temporal artery biopsy.
C. Platelet count.
D. Retinal photography.
E. Serum protein electrophoresis.

A
163
Q

RACP 2006a QUESTION 6
Which of the following factors is most predictive of future erosions in a patient with rheumatoid arthritis?
A. Sustained raised tender joint count.
B. Sustained prolonged early morning stiffness.
C. Sustained low serum C4 level.
D. Sustained raised C-reactive protein (CRP) level.
E. Expression of human lymphocyte antigen (HLA) - DR 3.

A
164
Q

RACP 2006a QUESTION 16
Which of the following cells is most likely to cause bony erosions in patients with rheumatoid arthritis?
A. T-Lymphocyte.
B. B-Lymphocyte.
C. Osteoclast.
D. Osteoblast.
E. Plasma cells.

A
165
Q

RACP 2006a QUESTION 34
Arthritis in which of the following joints is most suggestive of haemachromatosis?
A. Knee.
B. Ankle.
C. Distal interphalangeal.
D. Metacarpophalangeal.
E. Metatarsophalangeal.

A
166
Q

RACP 2006a QUESTION 49
The coupling of osteoblast and osteoclast function is mainly related to the action of which of the following factors?
A. Tumour Necrosis Factor (TNF).
B. Parathyroid hormone (PTH).
C. Receptor activator of NFKB (RANK) ligand.
D. 1,25 (OH)2 cholecalciferol.
E. Oestradiol.

A
167
Q

RACP 2006b QUESTION 8
A 25-year-old female presents with symmetrical small joint polyarthralgia associated with a
photosensitive facial rash, and low grade fever. Examination reveals small joint tenderness in the
hands without synovitis, and an erythematous, slightly scaly malar rash. The examination is otherwise
unremarkable, and urinalysis is normal. The following laboratory results are obtained.
C-reactive protein (CRP) 18 mg/L [0-11]
Antinuclear antibody (ANA) positive, titre: 1/2560; pattern: homogeneous
Anti-double stranded DNA (anti-dsDNA) 17 IU/mL [0-5]
Rheumatoid factor <20 IU/mL [0-20]
In addition to non-steroidal anti-inflammatory agents, which of the following is the most appropriate
next step in management?
A. Azathioprine.
B. Topical corticosteroids.
C. Methotrexate.
D. Oral corticosteroids.
E. Hydroxychloroquine.

A
168
Q

RACP 2006 b QUESTION 15
A 52-year-old woman presents with dry eyes which have progressively worsened over the past three years. On examination her Schirmer test is 1mm in 5 minutes [5mm in 5 minutes] and she has positive Rose Bengal staining of both corneas. Her erythrocyte sedimentation rate is 98 mm/hr
[<21mm/hr], antinuclear antibody is positive with a speckled pattern and a titre of 1:640 and Anti-Ro and –La antibodies are detected.
The most appropriate initial treatment is:
A. hydroxychloroquine.
B. low dose oral prednisolone.
C. methotrexate.
D. hypomellose tear substitute.
E. prednisolone eye drops.

A
169
Q

RACP 2006b QUESTION 24
A 65-year-old man developed Wegener’s granulomatosis eight years previously. At that time he had upper and lower respiratory tract and renal involvement. He achieved remission with prednis(ol)one and daily cyclophosphamide which was continued over 18 months. Seven months previously he
experienced a relapse with arthritis, sinusitis, an active renal sediment and a rise in serum creatinine from 90 to 115 μmol/L. The relapse was treated successfully with prednis(ol)one and weekly methotrexate. Prednis(ol)one was tapered and withdrawn. Currently, he feels well and examination is
normal. Laboratory tests show normal blood count, erythrocyte sedimentation rate (ESR) 25 mm/hr [0-20], and
normal creatinine. Urine examination shows no protein, 10-100 red cells/high power field, and there
are no dysmorphic cells.
Which of the following is the next most appropriate course of action?
A. Renal biopsy.
B. Replace methotrexate with cyclophosphamide.
C. Cystoscopy.
D. Serum antineutrophil cytoplasmic antibodies (ANCA) testing.
E. Reintroduce prednis(ol)one.

A
170
Q

RACP 2006b QUESTION 26
A 28-year-old woman presents with episodes of colour change in her distal fingers when they are exposed to the cold. The episodes occur most days and are characterised by white numb finger tips which become blue and then red. She describes similar episodes that have occurred for many years.
She is otherwise well. Examination is unremarkable. Investigations show a positive anti-nuclear antibody of a speckled pattern with a titre of 1:80. The most likely diagnosis is:
A. limited scleroderma.
B. diffuse scleroderma.
C. primary Raynaud’s phenomenon.
D. systemic lupus erythematosus.
E. carcinoid syndrome.

A
171
Q

RACP 2006b QUESTION 31
A 49-year-old female presents with a five week history of symmetrical polyarthralgia. The following
laboratory results are obtained:
Antinuclear antibody (ANA) positive, titre: 1/80, pattern: speckled
Antibodies to extractable nuclear antigens (ENAs) negative
Anti-double stranded DNA antibody (anti-dsDNA) 7 IU/mL [0-5]
Anti-cyclic citrullinated peptide (anti-CCP) positive
Anti-filaggrin antibody positive
Rheumatoid factor (RF) < 20 [0-20]
Which of the following is the most likely diagnosis?
A. Rheumatoid arthritis.
B. Systemic lupus erythematosus.
C. Mixed connective tissue disease.
D. Viral polyarthritis.
E. Osteoarthritis.

A
172
Q

RACP 2006b QUESTION 38
A 70-year-old woman has been on haemodialysis for 11 years. She complains of painful shoulders and knees and wrists and cannot raise her arms beyond 90°. She has noted a gradual deterioration in
her arthralgias over a period of 12 months. A radiograph of her wrist is shown below. Which of the following is most likely to explain her symptoms?

A. Pseudogout.
B. Proximal myopathy.
C. Dialysis related amyloidosis.
D. Gout.
E. Osteoarthritis.

A
173
Q

RACP 2006 B QUESTION 40
A 26-year-old man presents with a swollen, painful left ankle and right wrist ten days after being
treated with doxycycline for chlamydial urethritis. Which of the following is the most appropriate for the
initial treatment of his joint symptoms?
A. Azithromycin.
B. Prednisolone.
C. Sulfasalazine.
D. Indomethacin.
E. Hydroxychloroquine.

A
174
Q

RACP 2006b Q73
A 26-year-old man presents with a six month history of lower back and buttock pain with prolonged stiffness in the morning. Physical examination reveals a Schober test of 4 cm on flexion [>5cm].
Which of the following tests is most likely to support a diagnosis of ankylosing spondylitis?
A. X-ray sacroiliac joints.
B. HLA B27 genotyping.
C. Erythrocyte sedimentation rate.
D. Rheumatoid factor.
E. Radionuclide bone scan.

A

Answer ?A

175
Q

RACP 2006 b QUESTION 99

A 76-year-old woman presents with a swollen, painful left knee. The knee has become increasingly painful over two weeks and she is now unable to walk on it. She does not remember injuring the knee and is otherwise well. On examination she is afebrile and has a large left knee effusion with reduced range of movement. An X-ray of the knee is shown above. What is the most likely diagnosis?
A. Osteoarthritis.
B. Gout.
C. Pseudogout.
D. Haemarthrosis.
E. Septic arthritis.

A
176
Q

RACP 2005a Q4

Diabetic neuropathic arthropathy is most likely to affect which of the following areas?
A. Knee.
B. Mid-foot.
C. 1st metatarsophalangeal joint.
D. Elbow.
E. Ankle.

A
177
Q

RACP 2005a Q24

A 54-year-old man presents with neck pain. Examination reveals a limited range of movement of the neck. Which feature on the X-ray shown above best explains the limited range of movement?
A. Zygoapophyseal joint fusion.
B. Diffuse disc degeneration.
C. Upper cervical spine instability.
D. Calcification of the posterior longitudinal ligament.
E. Diffuse idiopathic skeletal hyperostosis.

A
178
Q

RACP 2005b QUESTION 4
An 84-year-old female nursing home resident is ambulant but rarely goes outside. She is thin, eats
little and has reflux oesophagitis. Medical history includes mastectomy and radiation therapy for breast cancer at age 60 years. There is a past history of deep venous thrombosis (DVT). Dual-energy X-ray absorptiometry (DEXA) scan reveals a t score of -2.2 at the spine and -3.0 at the femoral neck. Serum calcium is normal.
In addition to calcium supplementation, which of the following is the most appropriate initial therapy for her osteoporosis?
A. Alendronate.
B. Calcitonin.
C. Vitamin D.
D. Oestrogen.
E. Raloxifene.

A
179
Q

RACP 2005b QUESTION 17
An 82-year-old man presents with difficulty walking due to osteoarthritis in his right hip. When prescribing a cane as a walking aide for him, the most appropriate instruction regarding use of the cane is:
Hold the cane:
A. in the right hand and advance the cane with the right leg.
B. in the right hand and advance the cane with the left leg.
C. in the left hand and advance the cane with the right leg.
D. in the left hand and advance the cane with the left leg.
E. in the right hand and advance the cane after both feet have advanced.

A
180
Q

RACP 2005 Q22

A 52-year-old female presents with a 12-month history of painful swollen fingers. X-rays of her hands are shown above. For which of the following conditions are these findings most characteristic?
A. Rheumatoid arthritis.
B. Systemic lupus erythematosus.
C. Psoriatic arthritis.
D. Gout.
E. Scleroderma.

A
181
Q

RACP 2005b QUESTION 40
A 75-year-old woman has bilateral knee osteoarthritis, diabetes and ischaemic heart disease.
Functionally she can walk two kilometres with pain in her back usually causing her to stop. She
presents with a four-week history of severe pain in the right knee on weight bearing. This pain is
minimally improved with non-steroidal anti-inflammatory drugs, paracetamol and intra-articular corticosteroid injection, and there is no history of trauma or falls. On examination there is no heat but a small effusion, the knee is tender along the medial joint line and ligaments appear normal. Her X-
ray report suggests moderate changes consistent with osteoarthritis.

Which of the following investigations is most likely to provide the diagnosis?
A. Gallium scan.
B. Bone scan.
C. Computed tomography (CT) scan.
D. Magnetic resonance imaging (MRI) scan.
E. Arthroscopy.

A
182
Q

RACP 2005b QUESTION 50
A 60-year-old oyster-farmer presents with a six-week history of pain and swelling in his right hand. He
initially developed symptoms in the 3rd and 4th metacarpophalangeal (MCP) joints as well as the
dorsum of the wrist. He has occasionally cut his hands during his work and recalls having serous
drainage from a cut on his right hand two months previously.
Examination confirms tenosynovial swelling over the right wrist and soft tissue swelling of the 3rd and 4th MCP joints. There are no skin lesions or evidence of other joint pathology.
Blood count and chest X-ray are normal.
Following aspiration of fluid from the tenosynovial effusion, which of the following synovial fluid studies would be most likely to provide the correct diagnosis?
A. Synovial fluid rheumatoid factor.
B. Culture for acid fast organisms.
C. Culture for fungi.
D. Polymerase chain reaction (PCR) examination for virus particles.
E. Gram stain and culture for bacteria.

A
183
Q

RACP 2005b QUESTION 74
A 38-year-old woman presents with a six-week history of increasing shortness of breath while playing tennis and a two-week history of increasing muscle weakness. She reports mild symmetrical
polyarthralgia of the hands, wrists and knees for six months and Raynaud’s phenomenon during the
recent winter. Examination reveals proximal muscle weakness and a hyperkeratotic rash with
fissuring on the palmar and lateral surfaces of the fingers on both hands. Investigations reveal a
serum creatine kinase (CK) level of 890 U/L [16-139] and an interstitial pulmonary infiltrate on high resolution computed tomography (CT) scan of the chest.
Autoantibody tests show the following results:
anti-nuclear antibody (ANA) test titre 1:2560, speckled pattern
anti-ENA (extractable nuclear antigens) antibodies Jo-1 positive
anti-DNA antibodies negative
The most likely diagnosis is:
A. dermatomyositis.
B. progressive systemic sclerosis.
C. polymyositis (anti-synthetase syndrome).
D. systemic lupus erythematosus.
E. Sjögren’s syndrome.

A
184
Q

RACP 2005b QUESTION 88
A 54-year-old Indian woman complains of a constant pain in her hips which is worse on standing and
walking. She has mild weakness of hip flexion and walks with difficulty. She is a vegetarian and avoids dairy products. She immigrated to New Zealand three years ago. A pelvic X-ray is normal.
The following blood tests are obtained:
alkaline phosphatase (ALP) 457 IU/L [30-115]
corrected calcium 2.13 mmol/L [2.15-2.57]
phosphate 0.79 mmol/L [0.90-1.55]
parathyroid hormone (PTH) 16.0 pmol/L [1.3-7.6]
The most appropriate next investigation is:
A. Dual-energy X-ray absorptiometry (DEXA) scan.
B. bone biopsy.
C. isotope bone scan.
D. vitamin D levels.
E. 24-hour urinary calcium.

A
185
Q

RACP 2005b QUESTION 99
A 65-year-old woman presents with six months of worsening left knee pain. The knee pain is worse
during and after walking and improved by rest. On examination she has a body mass index (BMI) of
36 kg/m2
[18-25]. Her left knee is not swollen and has a painful but full range of movement.
Her X-ray is as shown below.

The most appropriate initial management, in addition to simple analgesia with paracetamol, is:
A. commence non-steroidal anti-inflammatory medications.
B. inject the left knee with a depot corticosteroid.
C. refer the patient for quadriceps muscle strengthening exercises.
D. refer to a dietician for a weight-loss program.
E. refer to an orthopedic surgeon for arthroscopy.

A
186
Q

RACP 2004a QUESTION 5
Knee pain experienced principally when walking downstairs is most likely to reflect pathology in which
of the following knee structures?
A. Medial compartment.
B. Lateral compartment.
C. Retropatellar compartment.
D. Suprapatellar bursa.
E. Lateral meniscus.

A
187
Q

RACP 2004a QUESTION 18

Which of the following conditions is most likely to be present in a patient with the nailfold lesions
shown above?
A. Polymyositis.
B. Systemic sclerosis.
C. Systemic lupus erythematosus (SLE).
D. Seropositive rheumatoid arthritis.
E. Multicentric reticulohistiocytosis.

A
188
Q

QUESTION 31
In a woman with antiphospholipid syndrome and recurrent pregnancy loss, which of the following treatment approaches provides the greatest chance of her achieving a successful pregnancy?
A. Low-dose corticosteroids.
B. Low-dose corticosteroids and aspirin.
C. Aspirin and low molecular weight heparin.
D. Intravenous immunoglobulin.
E. Warfarin.

A
189
Q

RACP 2004a QUESTION 60
In rheumatoid arthritis, the slow-acting or so-called ‘disease-modifying’ antirheumatic drugs have been shown convincingly to retard joint damage. Although in osteoarthritis it remains to be proven that any intervention alters the course of the disease, which of the following has the strongest experimental support in the clinical situation as a therapy able to retard cartilage loss?
A. Ketoprofen.
B. Glucosamine sulphate.
C. Rofecoxib.
D. Intra-articular corticosteroids.
E. Intra-articular hyaluran.

A
190
Q

RACP 2004b QUESTION 5
For which of the following conditions is walking exercise most efficacious compared with weightlifting?
A. Insulin resistance.
B. Depression.
C. Osteoarthritis of the knee.
D. Vascular claudication.
E. Falls.

A
191
Q

RACP 2004b QUESTION 30
A frail 88-year-old woman is admitted to hospital within a few hours of having developed sudden,
severe and disabling pain in the lower back and right buttock. She has previously been well aside
from well-controlled hypertension and the resection of a sigmoid colon cancer 15 years ago.
Examination is difficult because of pain. There is considerable but poorly localised tenderness around
the sacral area and buttock, pain on movement of either hip and no obvious neurological deficit. X-rays of the lumbo-sacral spine, pelvis and hips reveal degenerative changes in the lumbosacral spine of a degree expected in a woman of her age. A bone scan, performed on the afternoon of her admission, is shown below.

The most likely diagnosis is:
A. prolapsed intervertebral disc causing sciatic pain.
B. stress fracture of the sacrum or pelvis.
C. metastatic bone disease.
D. musculo-ligamentous low back strain.
E. referred pain from pelvic recurrence of colonic neoplasm.

A
192
Q

RACP 2004b QUESTION 60
An 83-year-old man has osteoarthritis of the knee. Pain limits his ability to walk more than 300 m.
Paracetamol gives minimal relief. Examination of the knee reveals a varus deformity and a warm effusion. He is commenced on naproxen 500 mg twice daily. Four weeks later his knee has not
improved and his serum creatinine has risen from 140 mmol/L to 350 mmol/L [60-120].
In addition to cessation of naproxen, which of the following would be the most appropriate next step in the management of his osteoarthritis of the knee?
A. Sulindac.
B. Rofecoxib.
C. Intra-articular corticosteroids.
D. Intra-articular hyaluronic acid.
E. Total joint replacement.

A
193
Q

RACP 2004b QUESTION 64
A 60-year-old Polynesian man presents with a 12-month history of pain and swelling in the left third proximal interphalangeal joint. X-rays of the hand and affected finger are shown below.

The X-rays are most consistent with which of the following?
A. Gout.
B. Tuberculous arthritis.
C. Chronic septic arthritis.
D. Hyperparathyroidism.
E. Psoriatic arthritis.

A
194
Q

RACP 2004b QUESTION 69
A 54-year-old Indian woman complains of a constant pain in her hips which is worse on standing and
walking. She has mild weakness of hip flexion and walks with difficulty. She is a vegetarian and avoids dairy products. She immigrated to New Zealand three years ago. A pelvic X-ray is normal. The following blood tests are obtained:
alkaline phosphatase (ALP) 457 U/L [30-115]
corrected calcium 2.13 mmol/L [2.15-2.57]
phosphate 0.79 mmol/L [0.90-1.55]
parathyroid hormone (PTH) 16.0 pmol/L [1.3-7.6]
The most appropriate next investigation is:
A. dual-energy X-ray absorptiometry (DEXA) scan.
B. bone biopsy.
C. isotope bone scan.
D. vitamin D levels.
E. 24-hour urinary calcium.

A
195
Q

RACP 2004b QUESTION 86
A previously well 58-year-old woman presents with a two-year history of marked sicca symptoms,
intermittent bilateral parotid swelling and an abnormal Schirmer’s test. Investigations reveal a positive antinuclear antibody (ANA) test (titre >1:2560), speckled pattern with positive Ro (Sjögren’s syndrome
A (SS-A)) and La (Sjögren’s syndrome B (SS-B)) antibody tests and negative anti-DNA antibodies. Rheumatoid factor is strongly positive and marked polyclonal gammopathy is noted on serum protein electrophoresis (EPG). Full blood count is normal. Serum biochemistry is normal except for a serum bicarbonate of 15 mmol/L [24-31].
In the long-term follow-up, which of the following features is least likely to occur?
A. Glomerulonephritis.
B. Raynaud’s phenomenon.
C. Arthritis.
D. Nephrocalcinosis
E. Lymphoma.

A

Answer ?D Nephrocalcinosis

Patient has primary Sjogrens syndrome with systemic involvement
- low bicarb is RTA type 1
- elevated globulins - increase risk factor for lymphoma
- All other options are systemic manifestations of Sjogrens except neprhocalcinosis

196
Q

QUESTION 94
A 45-year-old woman is referred following relocation from interstate. Her accompanying medical records list her major diagnosis as systemic lupus erythematosus (SLE). You elicit a history of three
to four years of polyarthritis, Raynaud’s phenomenon, myalgia and proximal muscular weakness and
recently, increasing shortness of breath on exertion. Examination reveals a blood pressure of 120/70 mmHg, a single prominent second heart sound, diffuse puffiness of the digits, mild proximal weakness and mild synovitis of the wrists and small joints of the hands. Which of the following would be the most likely cause of death in this woman?
A. Diffuse proliferative glomerulonephritis.
B. Myocarditis.
C. Renovascular hypertension.
D. Pulmonary hypertension.
E. Cerebral vasculitis.

A
197
Q

RACP 2004b QUESTION 98
An 85-year-old woman who fell in her home one week ago presents for follow-up examination.
At the time of her fall she experienced immediate pain in her buttocks and hips and was brought to the emergency department, where all X-rays of the hips and pelvis were negative for fracture and she was
sent home. A week later she continues to complain of severe pain and is unable to weight bear. On examination, she has tenderness in the buttock area but a good range of motion in the hip joints.
Which of the following is the most likely cause of her pain?
A. Sciatica.
B. Lumbar stenosis.
C. Ischial bursitis.
D. Osteoarthritis of the hip.
E. Pelvic fracture.

A
198
Q

RACP 2003a Q30

The condition shown in the X-rays above is most likely to be seen with which one of the following?
A. Haemochromatotic arthropathy.
B. Osteoarthritis secondary to trauma.
C. Primary osteoarthritis.
D. Haemophilic arthropathy.
E. Ochronotic arthropathy.

A
199
Q

RACP 2003a Q61

A 64-year-old woman undergoes a dual-energy X-ray absorptiometry scan. The report is shown below. According to World Health Organisation (WHO) definitions, the patient’s lumbar spine bone density is:
A. high normal.
B. normal.
C. osteopenic.
D. osteoporotic.
E. severely osteoporotic.

A
200
Q

RACP 2003b QUESTION 4
An 85-year-old woman who fell in her home one week ago presents for follow-up examination.
At the time of her fall she experienced immediate pain in her buttocks and hips and was brought to the emergency department, where all X-rays of the hips and pelvis were negative for fracture and she was sent
home. A week later she continues to complain of severe pain and is unable to weight bear. On examination, she
has tenderness in the buttock area but a good range of motion in the hip joints.
What is the most likely cause of her pain?
A. Sciatica.
B. Lumbar stenosis.
C. Ischial bursitis.
D. Osteoarthritis of the hip.
E. Pelvic fracture.

A
201
Q

RACP 2003b QUESTION 15
A 19-year-old man, who is sexually active and admits to occasional recreational drug use, presents with a
three-day history of severe pain over the left sacroiliac area. Examination reveals a temperature of 38°C,
pulse rate of 80/minute and blood pressure of 120/80 mmHg. The only other findings are ridged nails and
pain on movement of the spine or when pressure is applied to the sacroiliac joints.
Investigations show:
white cell count 12.5 x 109 /L [3.5-11.0]
haemoglobin normal
platelet count normal
erythrocyte sedimentation rate (ESR) 50 mm/h [<10]
C-reactive protein 170 mg/L [<10]
X-rays of the spine and pelvis are normal. The bone scan is shown below.

The most likely diagnosis is:
A. reactive arthritis following Chlamydia trachomatis infection.
B. psoriatic arthritis.
C. gonococcal arthritis.
D. septic sacroiliitis.
E. undifferentiated spondylarthropathy.

A
202
Q

RACP 2003b QUESTION 26
A 38-year-old woman presents with a six-week history of increasing shortness of breath while playing tennis
and a two-week history of increasing muscle weakness. She reports mild symmetrical polyarthralgia of the
hands, wrists and knees for six months and Raynaud’s phenomenon during the recent winter. Examination
reveals proximal muscle weakness and a hyperkeratotic rash with fissuring on the palmar and lateral
surfaces of the fingers on both hands. Investigations reveal a serum creatine kinase (CK) level of 890 U/L
[16-139] and an interstitial pulmonary infiltrate on high resolution computed tomography (CT) scan of the chest.
Autoantibody tests show the following results:
anti-nuclear antibody (ANA) test titre 1:2560, speckled pattern
anti-ENA (extractable nuclear antigens) antibodies Jo-1 positive
anti-DNA antibodies negative
The most likely diagnosis is:
A. dermatomyositis.
B. progressive systemic sclerosis.
C. polymyositis (anti-synthetase syndrome).
D. systemic lupus erythematosus.
E. Sjögren’s syndrome.

A
203
Q

RACP 2003b QUESTION 41
A 40-year-old man presents with a six-month history of Raynaud’s phenomenon and recurrent leg ulcers. Examination shows multiple small, palpable, purpuric skin lesions on both lower legs. Serum electrophoresis and immunoelectrophoresis shows an immunoglobulin M (IgM) paraprotein of 2 g/L and moderate
cryoglobulins with a cryocrit of 10%.
The most appropriate next investigation is:
A. anti-nuclear antibodies (ANA).
B. hepatitis C serology.
C. bone marrow biopsy.
D. skin biopsy.
E. rheumatoid factor.

A
204
Q

RACP 2003b QUESTION 83
A 24-year-old pregnant woman reports at her first antenatal visit that her sister gave birth to a child with
congenital heart block (CHB). Testing for which one of the following would be most useful in assessing the
risk of CHB in this pregnancy?
A. Fluorescent anti-nuclear antibodies (ANA).
B. Anti-DNA antibodies.
C. Antibodies to extractable nuclear antigens (ENA).
D. Anti-cardiolipin antibodies.
E. Lupus anticoagulant.

A
205
Q

RACP 2002a QUESTION 6
For an 82-year-old nursing home resident who experiences recurrent falls, which one of the following is least
likely to prevent hip fracture?
A. Oestrogen replacement therapy.
B. Bisphosphonates.
C. Calcium supplements.
D. Balance and muscle strengthening exercises.
E. Hip protectors.

A
206
Q

RACP 2002b QUESTION 15
A 70-year-old woman, weighing 45 kg, presents with polymyalgia rheumatica and headache. There is a
history of osteoporosis with a previous vertebral wedge fracture. Past history also includes hysterectomy.
Temporal artery biopsy reveals giant cell arteritis. A bone mineral density test shows T scores for the lumbar spine and the femoral neck as -3.0 and -2.5 respectively.
A chest X-ray shows wedging of the thoracic vertebra on the lateral view.
The most appropriate initial management is:
A. prednis(ol)one 40 mg daily and alendronate 10 mg daily.
B. prednis(ol)one 10 mg daily and azathioprine 100 mg daily.
C. prednis(ol)one 40 mg twice daily and calcitriol 0.25 μg twice daily.
D. prednis(ol)one 15 mg daily and methotrexate 10 mg weekly.
E. prednis(ol)one 40 mg daily and hormone replacement therapy.

A
207
Q

RACP 2002b QUESTION 21
The presence of antibodies to which one of the following extractable nuclear antigens (ENA) is most specific
for systemic lupus erythematosus?
A. Sm.
B. Ribonucleoprotein (RNP).
C. Ro (SS-A).
D. La (SS-B).
E. Jo-1.

A
208
Q

RACP 2002b QUESTION 28
A 45-year-old diabetic man presents with ‘arthritis in his hands’. X-rays of the left hand are shown below.

The most likely diagnosis is:
A. gout.
B. primary osteoarthritis.
C. hyperparathyroidism.
D. haemochromatosis.
E. rheumatoid arthritis.

A
209
Q

RACP 2002b QUESTION 36
A 24-year-old pregnant woman reports at her first antenatal visit that her sister gave birth to a child with
congenital heart block (CHB). Which one of the following tests would be most useful in assessing the risk of
CHB in this pregnancy?
A. Fluorescent antinuclear antibody (ANA) test.
B. Antibodies to DNA.
C. Antibodies to extractable nuclear antigens (ENA).
D. Anticardiolipin antibodies.
E. Lupus anticoagulant.

A
210
Q

RACP 2002b Q50
A 38-year-old woman presents with a six-week history of increasing shortness of breath on playing tennis and a two-week history of increasing muscle weakness. On system review she reports mild symmetrical polyarthralgia of the hands, wrists and knees for six months and Raynaud’s phenomenon during the recent
winter. Examination reveals proximal muscle weakness and a hyperkeratotic rash with fissuring on the
palmar and lateral surfaces of the fingers on both hands. Abnormal results include a high creatine kinase
(CK) level of 890 U/L [16-139] and an interstitial pulmonary infiltrate on high resolution computed
tomography (CT) scan of the chest. Autoantibody tests show the following results:
antinuclear antibody (ANA) test titre 1:2560, speckled pattern
anti-ENA (extractable nuclear antigens) antibodies Jo-1 positive
anti-DNA antibodies negative
The most likely diagnosis is:
A. dermatomyositis.
B. progressive systemic sclerosis.
C. polymyositis (antisynthetase syndrome).
D. systemic lupus erythematosus.
E. Sjögren’s syndrome.

A

Answer C (or A?)

Features of IIM and antisynthetase syndrome:
Proximal weakness with elevated CK
Mechanics hands - (hyperkeratanising with fissures)
ILD
Positive anti-Jo-1 = antisynthetase syndrome
Speckled ANA - seen in
SLE, Sjögren’s syndrome
Polymyositis/dermatomyositis, Systemic sclerosis
Neg Anti DNA - not SLE

PM: no cutaneous features of dermatomyositis

211
Q

RACP 2002b QUESTION 55
A 37-year-old woman who has had rheumatoid arthritis for five years attends for follow-up. She is currently
on treatment with methotrexate 7.5 mg once weekly, folic acid 10 mg once weekly, ketoprofen 200 mg daily
and prednis(ol)one 7.5 mg daily. She says that she is doing ‘very well’, with little pain or morning stiffness
and that her function is much better than it was two years previously. Examination reveals soft tissue
swelling in the wrists and most metacarpophalangeal (MCP) and proximal interphalangeal (PIP) joints with
subluxation of the 2nd MCP joint bilaterally.
Blood investigations show:
haemoglobin 105 g/L [115-165]
erythrocyte sedimentation rate (ESR) 45 mm/h [0-15]
C-reactive protein 42 mg/L [<10]
X-rays taken two years ago showed early erosions in the right 2nd MCP joint and the left carpus. The
current X-ray of her right hand is shown below.
The most appropriate treatment would be to:
A. add leflunomide (Arava) to current therapy.
B. increase the methotrexate dose.
C. replace methotrexate with cyclosporin.
D. reduce the prednis(ol)one dose to 5 mg daily.
E. temporarily increase the prednis(ol)one dose to 10 mg twice daily.

A
212
Q

RACP 2002b Q71
Which one of the following clinical features best differentiates inclusion body myositis from polymyositis?
A. Quadriceps wasting.
B. Deltoid wasting.
C. Weakness of long finger flexors.
D. Bilateral ptosis.
E. Truncal weakness.

A

Answer C Finger flexor weakness

Polymyositis (and other inflammatory myopathies) have symmetric proximal weakness

IBM - asymmetrical, distal and proximal weakness esp. finger flexors and quadriceps

213
Q

RACP 2002b QUESTION 83
A 64-year-old man with a history of recurrent gout and a recent past history of pulmonary embolism for which
he is taking warfarin (prothrombin time−international normalised ratio (PT-INR) 2.5 [1.0-1.3]), presents with
acute arthritis in the left mid-foot which is consistent with his previous attacks of crystal synovitis. Recently,
allopurinol was added to his drug therapy and the dose has been slowly increased to 200 mg daily. His
serum urate on admission is 0.49 mmol/L [0.18-0.45] and his serum creatinine is within the range of 0.16-0.18 mmol/L [0.05-0.13].
Which one of the following management options would be most appropriate for the treatment of the
synovitis?
A. Cease the allopurinol and add colchicine 0.5 mg three times daily.
B. Continue allopurinol at the current dose and add indomethacin 50 mg sixth hourly.
C. Increase the allopurinol to 300 mg daily and add prednis(ol)one 15 mg daily.
D. Reduce the allopurinol to 100 mg daily and add celecoxib 200 mg twice daily.
E. Continue allopurinol at the current dose and add prednis(ol)one 20 mg daily.

A
214
Q

RACP 2002b QUESTION 91
Which one of the following is the most common manifestation of Behçet’s disease?
A. Recurrent oral ulceration.
B. Recurrent genital ulceration.
C. Central nervous system symptoms.
D. Anterior uveitis.
E. Positive pathergy test.

A
215
Q

RACP 2001a Question 35

Which one of the following is the most common association of the abnormality shown in the X-ray above?

A. Haemochromatosis.

B. Hypomagnesaemia.

C. Old age.

D. Hyperparathyroidism.

E. Osteoporosis.

A
216
Q

RACP 2001b Q10

A 25-year-old man presents with a five-year history of low back pain and stiffness. Symptoms are generally worse in the morning with some improvement following activity. Over the past three years he has also had episodes of Achilles tendonitis. Examination reveals marked limitation of lumbar spine movements in all planes.
Investigations shown (in picture)

X-rays of the patient’s pelvis and lumbar spine are shown above.
What is the most appropriate initial management?

A. Paracetamol 1 g qid.
B. Indomethacin 25 mg tds.
C. Indomethacin 25 mg tds and a physical therapy programme.
D. A physical therapy programme and hydrotherapy.
E. Sulphasalazine 1 g bd and hydrotherapy.

A
217
Q

RACP 2001b Question 23

A patient being investigated for recurrent bilateral parotid swelling and xerostomia complains of generalised muscle weakness. Blood pressure, blood count, liver function tests and creatine kinase level are all normal. The serum potassium is 2.1 mmol/L [3.5-5.2] and the serum bicarbonate is 16 mmol/L [22-30].

Which one of the following is most likely to be diagnostic?

A. Thyroid function tests.

B. Antibodies to nuclear and extractable nuclear antigens (ANA and ENA).

C. Serum angiotensin-converting enzyme (ACE).

D. Acetylcholine receptor (AChR) antibodies.

E. Chest X-ray.

A
218
Q

RACP 2001b Question 38

A 25-year-old man presents with a history of three episodes of low back pain over six months, each lasting for ten days. The pain is worse later in the day, eased by rest, and is not associated with morning back stiffness.

Investigations show:
FBC - within normal limits
ESR - 19 (2-20 mm/h)
HLAB27 positive
X ray lumbosacral spine and sacroiliac joints - normal

The most likely diagnosis is:

A. non-specific low back pain.
B. spondylolysis.
C. discitis.
D. ankylosing spondylitis.
E. Scheuermann’s osteochondritis.

A
219
Q

RACP 2001b Question 66

A 64-year-old man with a history of recurrent gout and a recent past history of pulmonary embolism for which he is taking warfarin (prothrombin time-international normalised ratio (PT-INR) 2.5 [1.0-1.3]), presents with acute arthritis in the left mid-foot which is consistent with his previous attacks of crystal synovitis. Recently, allopurinol was added to his other drug therapy and the dose has been slowly increased to 200 mg/day. His serum urate on admission is 0.49 mmol/L [0.18-0.45] and his creatinine is within the range of 0.16-0.18 mmol/L [0.05-0.13].

Which one of the following management options would be most appropriate for the treatment of the synovitis?

A. Cease the allopurinol and add colchicine 0.5 mg tds.
B. Continue allopurinol at the current dose and add indomethacin 50 mg 6th hourly.
C. Increase the allopurinol to 300 mg/day and add prednis(ol)one 15 mg/day.
D. Reduce the allopurinol to 100 mg/day and add celecoxib 200 mg bd.
E. Continue allopurinol at the current dose and add prednis(ol)one 20 mg/day.

A

Answer E

Avoid changes to urate lowering therapy (allopurinol) during acute gout flare - can prolong or worsen the flare.

Acute management:
NSAIDs - contraindicated here due to deranged renal function OR
Prednisone - suitable OR
Colchicine - also needs renal adjustment - 0.5 mg TDS is incorrect dose

220
Q

RACP 2001b Question 80

A 56-year-old woman with longstanding rheumatoid arthritis presents with a three-month history of increasing groin pain radiating to the knees. Her magnetic resonance imaging (MRI) scan is shown below.

The most likely cause of the pain is:

A. synovitis.

B. osteoarthritis.

C. avascular necrosis.

D. pigmented villonodular synovitis.

E. psoas abscess

A
221
Q

RACP 2001b Question 100

A 68-year-old woman with longstanding rheumatoid arthritis presents with a four-month history of worsening occipital headache. She has advanced peripheral joint damage and a restricted range of movement of the cervical spine. There is mild proximal and distal lower limb weakness with hyper-reflexia in both the upper and lower limbs. The plantar responses are flexor.

Investigations show:
Hb 108 [120-160]
ESR 105 [2-20]
WBC count normal
Platelet count normal

Liver function tests are normal except for a serum alkaline phosphatase level of 146 U/L [30-115].

What is the next most appropriate investigation?

A. Lateral view X-ray of the cervical spine with flexion and extension views.

B. Cervical magnetic resonance imaging (MRI) scan.

C. Temporal artery biopsy.

D. Upper limb electromyography.

E. Bone scan.

A
222
Q

RACP 2000a Question 19

Which one of the following drugs, when used continuously in the treatment of Paget’s disease, is most likely to cause defective mineralisation of newly formed bone matrix?

A. Pamidronate.

B. Calcitonin.

C. Mithramycin.

D. Alendronate.

E. Etidronate.

A
223
Q

RACP 2000a Question 56

A 68-year-old man with rheumatoid arthritis for 15 years has the following bone densitometry results on dual energy X-ray absorptiometry scan:

The most likely explanation for the discrepancy between the femoral neck and lumbar spine readings is:

A. technical error.

B. Paget’s disease in lumbar spine.

C. lumbar spondylosis.

D. secondary osteoarthritis in the hip.

E. synovitis in the hip

A
224
Q

RACP 2000b Question 4

A 25-year-old man presents with a history of three episodes of low back pain over six months, each lasting for ten days. The pain was worse later in the day, eased by rest, and was not associated with morning stiffness in the back.

Investigations show:
FBC - within normal limits
ESR 19 [2-20]
HLAB27 positive

X-ray of lumbosacral spine and sacro-iliac joints normal

The most likely diagnosis is:

A. non-specific low back pain.

B. spondylolysis.

C. discitis.

D. ankylosing spondylitis.

E. Scheuermann’s osteochondriti

A
225
Q

RACP 2000a Question 13

A 22-year-old woman presents with an eight-week history of erythematous rash on the arms and malar region, mild arthralgia and malaise.

Investigations show:

The most appropriate initial treatment is:

A. paracetamol.

B. naproxen.

C. hydroxychloroquine.

D. prednis(ol)one 15 mg/day.

E. pulse cyclophosphamide.

A
226
Q

RACP 2000b Question 42

A 38-year-old woman presents with pain over the radial side of her right wrist radiating down into the thumb. There is no swelling but she is locally tender just distal to the radial styloid. The pain is reproduced by ulnar deviation of the wrist with the thumb folded across the palm.

The most likely diagnosis is:

A. osteoarthritis of the first carpometacarpal joint.

B. avascular necrosis of the scaphoid.

C. rheumatoid arthritis.

D. calcium pyrophosphate deposition disease.

E. de Quervain’s tenosynovitis.

A
227
Q

RACP 2000b Question 60

A 43-year-old man with rheumatoid arthritis, inadequately controlled on ibuprofen and methotrexate (15 mg/week), is commenced on cyclosporin 3 mg/kg/day. His baseline serum creatinine is 0.05 mmol/L [0.05-0.09] and his blood pressure is 130/80 mmHg.

Six weeks later, his serum creatinine is 0.09 mmol/L and his blood pressure is 150/95 mmHg.

The most appropriate next step in management is to:

A. add nifedipine.

B. add diltiazem.

C. cease ibuprofen.

D. cease methotrexate.

E. cease cyclosporin.

A
228
Q

RACP 2000 Question 68

A 58-year-old man presents with a two-month history of knee and wrist pain. He has mild local tenderness with no soft tissue swelling or joint effusion. The physical examination is otherwise unremarkable.

An X-ray of his knee is shown below.

Which one of the following is the most appropriate next investigation?

A. Chest X-ray.

B. Joint aspiration.

C. Isotope bone scan.

D. Serum rheumatoid factor.

E. Serum calcium and parathyroid hormone

A
229
Q

RACP 2000b Question 77

A 72-year-old man with longstanding diabetes presents with a three-month history of discomfort and swelling in the midtarsal region.

Investigations show:
FBC normal
ESR 22 [2-14]
EUC normal
Serum Uric acid 0.49 [0.20-0.47 mmol/L]
Glycosylated haemoglobin 7.9% [<6.1%]
RF 22U/mL [0-20]

The X-ray of his foot is shown below.

The most likely cause of the foot pain is:

A. osteoarthritis.
B. gout.
C. septic arthritis.
D. neuropathic arthropathy.
E. rheumatoid arthritis.

A
230
Q

RACP 2000b Question 87

A 44-year-old man presents with a four-day history of acute right knee pain. He has a 10-year history of episodic pain and swelling of his toes, ankles and wrists. Examination reveals a warm, swollen right knee with an effusion. There is soft tissue swelling in both wrists, the left elbow and the right ankle and forefoot. A previous X-ray of his foot is shown below.

Investigations show:
Hb 150 [130-180]
WCC 12 x10 ^9 [4-11]
ESR 90[1-10]
CRP 186 [0-8]

Which one of the following is the most likely diagnosis?

A. Rheumatoid arthritis.
B. Gout.
C. Chondrocalcinosis.
D. Seronegative arthritis.
E. Septic arthritis.

A
231
Q

RACP 2000b Question 93

A 62-year-old woman with a 15-year history of rheumatoid arthritis presents with a four-month history of intermittent occipital headaches. She has mildly active synovitis. Neurological examination is normal.

The next most appropriate investigation is:

A. computed tomography (CT) scan of the brain.

B. skull X-ray.

C. magnetic resonance imaging (MRI) scan of the brain.

D. lateral flexion X-ray of cervical spine.

E. computed tomography (CT) scan of the neck.

A
232
Q

RACP 2000b Question 97

A 64-year-old woman has painful swelling, pallor and blue discolouration of her fingers when cold.

An X-ray of one of her hands is shown below.

Which one of the following autoantibodies is most likely to be present?

A. Anti-RNP.

B. Anti-Pm-Scl.

C. Anti-centromere.

D. Anti-topoisomerase 1 (Scl 70).

E. Anti-filaggarin.

A
233
Q

RACP 1999a Question 7

Which one of the following is least likely to be associated with a flare of systemic lupus erythematosus?

A. Falling level of C3.

B. Falling peripheral blood leucocyte count.

C. Increasing titre of antibodies to double-stranded DNA.

D. Increasing peripheral blood platelet count.

E. Increasing urinary protein level.

A
234
Q

RACP 1999a A 68-year-old man with rheumatoid arthritis for 15 years has the following bone densitometry results on dual energy X-ray absorptiometry scan:

The most likely explanation for the discrepancy between the femoral neck and lumbar spine readings is:

A. technical error.

B. Paget’s disease in lumbar spine.

C. lumbar spondylosis.

D. secondary osteoarthritis in the hip.

E. synovitis in the hip

A
235
Q

RACP 2000a Question 51

Which one of the following is of least prognostic value in early rheumatoid arthritis?

A. Radiological evidence of erosions.

B. Rheumatoid factor titre.

C. Peripheral blood lymphocyte count.

D. C-reactive protein level.

E. Presence of rheumatoid nodules.

A
236
Q

RACP 1999b PAPER 2

Question 3

A 25-year-old woman presents with a three-month history of aching and stiffness of her joints. She also reports fatigue and Raynaud’s phenomenon. Her only medication is minocycline for acne.

The appearance of the patient’s hands is shown.

The following blood results are obtained:

Which of the following is the most likely diagnosis?

A. Rheumatoid arthritis.

B. Systemic lupus erythematosus.

C. Drug induced lupus erythematosus.

D. Mixed connective tissue disease.

E. Systemic sclerosis.

A
237
Q

RACP 1999b Question 6

A 37-year-old woman presents with a two-week history of pain in the region of the right hip. She has a five-year history of systemic lupus erythematosus with diffuse proliferative glomerulonephritis which was treated initially with high dose prednis(ol)one (1 mg/kg/day), but has recently been controlled by hydroxychloroquine 200 mg/day and prednis(ol)one 7.5 mg/day.

Examination of the right hip reveals moderate painful restriction of movement without local tenderness. Other joints are not painful or inflamed.

X-ray of the right hip shows no abnormality. A technetium 99m MDP bone scan of the right hip is shown below.

Which one of the following is the most likely diagnosis?

A. Acute synovitis.

B. Septic arthritis.

C. Osteoarthritis.

D. Femoral neck fracture.

E. Avascular necrosis.

A
238
Q

RACP 1999b Question 10

A 42-year-old woman with severe progressive systemic sclerosis presents with a three-week history of malaise, severe headaches and dyspnoea. Examination findings include a blood pressure of 180/115 mmHg, cardiomegaly, basal lung crepitations, retinal haemorrhages, and widespread skin tethering.

Investigations show:

The most appropriate initial treatment is:

A. pulse intravenous methyl prednis(ol)one.

B. plasmapheresis.

C. frusemide.

D. captopril.

E. renal dialysis.

A
239
Q

RACP 1999b Question 24

A previously well 45-year-old man presents with pain, swelling and redness in the left first metatarsophalangeal joint, and a diffusely swollen and painful left fourth toe. He has also had several episodes of low back pain, settling after several weeks, and a recent episode of diarrhoea. Investigations show:
FBC normal
ESR 28 [<20]
Serum uric acid 0.49 [0.20-0.47]
RF negative

The most likely diagnosis is:

A. Seronegative rheumatoid arthritis.
B. Polyarticular gout.
C. Ankylosing spondylitis.
D. Reactive arthritis.
E. Adult onset Still’s disease

A
240
Q

RACP 1999b Question 83

A 72-year-old woman has rheumatoid arthritis treated with prednis(ol)one 5 mg per day and methotrexate 12.5 mg per week. She is admitted to hospital because of cellulitis in her leg, and treated with intravenous flucloxacillin.

The last dose of methotrexate was on the day before admission.

Investigations on admission show:

The most appropriate additional treatment at this stage is:

A. none.

B. increase prednis(ol)one.

C. granulocyte colony stimulating factor.

D. folic acid.

E. folinic acid (leucovorin calcium).

A